SlideShare a Scribd company logo
1 of 47
NUR-634 Final Exam Test Prep
· You are examining an elderly man and notice the following:
Decreased vibration sense in the feet and ankles, diminished
gag reflex, right patellar reflex less than the left, and
diminished abdominal reflexes. Which of these is abnormal?
· Decreased vibration sense
· Diminished gag reflex
· Diminished right patellar reflex compared to the left
· Diminished abdominal reflexes
· Question Points: 1.0 / 1.0
·
A 68-year-old retired farmer presents to your office for
evaluation of a skin lesion. On the right temporal area of the
forehead, you see a flattened papule the same color as his skin,
covered by a dry scale that is round and feels hard. He has
several more of these scattered on the forehead, arms, and legs.
Based on this description, what is your most likely diagnosis?
· Actinic keratosis
· Seborrheic keratosis
· Basal cell carcinoma
· Squamous cell carcinoma
· Question Points: 1.0 / 1.0
· A 14-year-old junior high school student is brought in by his
mother and father because he seems to be developing breasts.
The mother is upset because she read on the Internet that
smoking marijuana leads to breast enlargement in males. The
young man adamantly denies using any tobacco, alcohol, or
drugs. He has recently noticed changes in his penis, testicles,
and pubic hair pattern. Otherwise, his past medical history is
unremarkable. His parents are both in good health. He has two
older brothers who never had this problem. On examination, you
see a mildly overweight teenager with enlarged breast tissue
that is slightly tender on both sides. Otherwise his examination
is normal. He is agreeable to taking a drug test.
What is the most likely cause of his gynecomastia?
· Breast cancer
· Imbalance of hormones of puberty
· Drug use
· Alcohol use
· Question Points: 1.0 / 1.0
· A 17-year-old female presents to your office, complaining of a
clear discharge from her right breast for 2 months. She states
that she noticed it when she and her boyfriend were “messing
around” and he squeezed her nipple. She continues to have this
discharge anytime she squeezes that nipple. She denies any
trauma to her breasts. Her past medical history is unremarkable.
She denies any pregnancies. Both of her parents are healthy.
She denies using tobacco or illegal drugs and drinks three to
four beers a week. On examination, her breasts are symmetric
with no skin changes. You are able to express clear discharge
from her right nipple. You feel no discrete masses and her
axillae are normal. The remainder of her heart, lung, abdominal,
and pelvic examinations are unremarkable. A urine pregnancy
test is negative.
What cause of nipple discharge is the most likely in her
circumstance?
· Benign breast abnormality
· Breast cancer
· Nonpuerperal galactorrhea
· Pagets disease
· Question Points: 1.0 / 1.0
· A 26-year-old flight attendant presents in for a third trimester
prenatal visit. She has had prenatal care since her sixth week of
pregnancy. She has no complaints today and her prenatal course
has been unremarkable. Today her blood pressure and weight
gain are appropriate, and her urine is unremarkable. You have a
first-year medical student shadowing you, so you ask the
student to get Doptones and measure the patient's uterus in
centimeters. The nurse practitioner student promptly reports
fetal heart tones of 140, but he is having difficulty obtaining the
correct measurement. He knows one end of the tape goes over
the uterine fundus.
From what inferior anatomic position should the tape be placed?
· Vagina
· Clitoris
· Pubic symphysis
· Umbilicus
· Question Points: 1.0 / 1.0
· A 58-year-old gardener presents to your office for evaluation
of a new lesion on her upper chest. The lesion appears to be
“stuck on” and is oval, brown, and slightly elevated with a flat
surface. It has a rough, wart like texture on palpation. Based on
this description, what is your most likely diagnosis?
· Actinic keratosis
· Seborrheic keratosis
· Basal cell carcinoma
· Squamous cell carcinoma
· Question Points: 0.0 / 1.0
· Which of the following represents age-related changes in the
lungs?
· Decrease in chest wall compliance
· Speed of expiration increases
· Increase in respiratory muscle strength
· Increased elastic recoil of lung tissue
· Question Points: 1.0 / 1.0
Top of Form
· A 15-month-old is brought to you for a fever of 38.6 degrees
Celsius and fussiness. The ear examination is as follows:
external ear, normal appearance and no tenderness with
manipulation; canal, normal diameter without evidence of
inflammation; tympanic membrane, bulging, erythematous, and
opaque. Insufflation is deferred due to pain. What is the most
likely condition here?
· Otitis externa
· Cholesteatoma
· Ruptured tympanic membrane
· Otitis media
· Question Points: 1.0 / 1.0
Bottom of Form
· A 28-year-old musician presents to your clinic, complaining of
a “spot” on his penis. He states his partner noticed it 2 days ago
and it hasn't gone away. He says it doesn't hurt. He has had no
burning with urination and no pain during intercourse. He has
had several partners in the last year and uses condoms
occasionally. His past medical history consists of
nongonococcal urethritis from Chlamydia and prostatitis. He
denies any surgeries. He smokes two packs of cigarettes a day,
drinks a case of beer a week, and smokes marijuana and
occasionally crack. He has injected IV drugs before but not in
the last few years. He is single and currently unemployed. His
mother has rheumatoid arthritis, and he doesn't know anything
about his father. On examination, you see a young man
appearing deconditioned but pleasant. His vital signs are
unremarkable. On visualization of his penis, there is a 6-mm
red, oval ulcer with an indurated base just proximal to the
corona. There is no prepuce because of neonatal circumcision.
On palpation, the ulcer is nontender. In the inguinal region,
there is nontender lymphadenopathy.
What disorder of the penis is most likely the diagnosis?
· Condylomata acuminata
· Genital herpes
· Syphilitic chancre
· Penile carcinoma
· Question Points: 1.0 / 1.0
· A 22-year-old law student presents to your office complaining
of severe abdominal pain radiating to his back. He states it
began last night after hours of heavy drinking. He has had
abdominal pain and vomiting in the past after drinking but
never as bad as this. He cannot keep any food or water down,
and these symptoms have been going on for almost 12 hours. He
has had no recent illnesses or injuries. His past medical history
is unremarkable. He denies smoking or using illegal drugs but
admits to drinking 6 to 10 beers per weekend night. He admits
that last night he drank something like 14 drinks. On
examination you find a young male appearing his stated age in
some distress. He is leaning over on the examination table and
holding his abdomen with his arms. His blood pressure is 90/60
and his pulse is 120. He is afebrile. His abdominal examination
reveals normal bowel sounds, but he is very tender in the left
upper quadrant and epigastric area. He has no Murphy's sign or
tenderness in the right lower quadrant. The remainder of his
abdominal examination is normal. His rectal, prostate, penile,
and testicular examinations are normal. He has no inguinal
hernias or tenderness with that examination. Blood work is
pending.
What etiology of abdominal pain is most likely causing his
symptoms?
· Peptic ulcer disease
· Biliary colic
· Acute cholecystitis
· Acute pancreatitis
· Question Points: 1.0 / 1.0
· A 56-year-old female presents to your clinic complaining that
her left breast looks unusual. She says that for 2 months the
angle of the nipple has changed direction. She does not do self-
examinations, so she doesn't know if she has a lump. She has no
history of weight loss, weight gain, fever, or night sweats. Her
past medical history is significant for high blood pressure. She
smokes two packs of cigarettes a day and has three to four
drinks per weekend night. Her paternal aunt died of breast
cancer in her forties. Her mother is healthy, but her father died
of prostate cancer. On examination, you find a middle-aged
woman appearing older than her stated age. Inspection of her
left breast reveals a flattened nipple deviating toward the lateral
side. On palpation, the nipple feels thickened. Lateral to the
areola you palpate a nontender 4-cm mass. The axilla contains
several fixed nodes. The right breast and axilla examinations
are unremarkable.
What visible skin change of the breast does she have?
· Nipple retraction
· Paget's disease
· Peau d'orange sign
· Mastitis
· Question Points: 0.0 / 1.0
· A 32-year-old white female presents to your clinic
complaining of overwhelming sadness. She says for the past 2
months she has had crying episodes, difficulty sleeping, and
problems with overeating. She says she used to go out with her
friends from work, but now she just wants to go home and be by
herself. She also thinks that her work productivity has been
dropping because she just is too tired to care or concentrate.
She denies any feelings of guilt or any suicidal ideation. She
states that she has never felt this way in the past. She denies
any recent illness or injuries. Her past medical history consists
of an appendectomy when she was a teenager; otherwise, she
has been healthy. She is single and works as a clerk in a
medical office. She denies tobacco, alcohol, or illegal drug use.
Her mother has high blood pressure and her father has had a
history of mental illness. On examination, you see a woman
appearing her stated age who seems quite sad. Her facial
expression does not change while you talk to her and she makes
little eye contact. She speaks so softly you cannot always
understand her. Her thought processes and content seem
unremarkable.
What type of mood disorder do you think is most likely?
· Dysthymic disorder
· Manic (bipolar) disorder
· Major depressive episode
· Anxiety disorder
· Question Points: 1.0 / 1.0
· A young Hispanic mother brings in her 2-month-old son. She
is upset because her neighbors have threatened to call the Child
Protective Agency because they think his birthmark is a bruise.
Her son was the product of an uneventful pregnancy and
spontaneous vaginal delivery. On examination, you see a large,
smooth-bordered bluish mark on his buttock and lower back.
Otherwise his examination is unremarkable.
What form of birthmark is this likely to be?
· Café-au-lait spot
· Salmon patch
· Mongolian spot
· Cherry hemangioma
· Question Points: 1.0 / 1.0
· A 15-year-old high school sophomore presents to the clinic for
evaluation of a 3-week history of sneezing; itchy, watery eyes;
clear nasal discharge; ear pain; and nonproductive cough.
Which is the most likely pathologic process?
· Infection
· Inflammation
· Allergic
· Vascular
· Question Points: 1.0 / 1.0
· A 22-year-old clerk, primigravida, presents to your office for
a prenatal visit. She is in her second trimester and has had
prenatal care since she was 8 weeks pregnant. Her only
complaint is that she has a new brownish line straight down her
abdomen. On examination her vital signs are unremarkable. Her
urine has no protein, glucose, or leukocytes. With a doppler the
fetal heart rate is 140, and her uterus is palpated to the
umbilicus. Today you are sending her for congenital
abnormality screening and setting up an ultrasound.
What physical finding is responsible for her new “brown line”?
· Corpus luteum
· Linea nigra
· Linea alba
· Diastasis recti
· Question Points: 1.0 / 1.0
· Mr. Q. is a 45-year-old salesman who presents to your office
for evaluation of fatigue. He has come to the office many times
in the past with a variety of injuries, and you suspect that he has
a problem with alcohol. Which one of the following questions
will be most helpful in diagnosing this problem?
· You are an alcoholic, aren't you?
· When was your last drink?
· Do you drink 2 to 3 beers every weekend?
· Do you drink alcohol when you are supposed to be working?
· Question Points: 1.0 / 1.0
· A 35-year-old bus driver presents to your office for a prenatal
visit. She is approximately 28 weeks pregnant and has had no
complications. She is complaining only of heartburn and has
had no fatigue, headaches, leg swelling, contractions, leakage of
fluid, or bleeding. On examination, her blood pressure is 142/92
and her urine shows no glucose, protein, or leukocytes. Her
weight gain is appropriate, with no large recent increases. Fetal
tones are 140 and her uterus measures 32 cm from the pubic
bone. Looking back through her chart, you see her prenatal
blood pressure was 120/70 and her blood pressures during the
first 20 weeks were usually 120 to 130/70 to 80.
What type of blood pressure is this?
· Normotensive for pregnancy
· Chronic hypertension
· Gestational hypertension
· Preeclampsia
· Question Points: 1.0 / 1.0
· A sudden, painless unilateral vision loss may be caused by
which of the following?
· Retinal detachment
· Corneal ulcer
· Acute glaucoma
· Uveitis
· Question Points: 0.0 / 1.0
· A 40-year-old mother of two presents to your office for
consultation. She is interested in knowing what her relative
risks are for developing breast cancer. She is concerned because
her sister had unilateral breast cancer 6 years ago at age 38. The
patient reports on her history that she began having periods at
age 11 and has been fairly regular ever since, except during her
two pregnancies. Her first child arrived when she was 26 and
her second at age 28. Otherwise, she has had no health
problems. Her father has high blood pressure. Her mother had
unilateral breast cancer in her 70s. The patient denies tobacco,
alcohol, or drug use. She is a family law attorney and is
married. Her examination is essentially unremarkable.
Which risk factor of her personal and family history most puts
her in danger of getting breast cancer?
· First-degree relative with premenopausal breast cancer
· Age at menarche of less than 12
· First live birth between the ages of 25 and 29
· First-degree relative with postmenopausal breast cancer
· Question Points: 1.0 / 1.0
· When should a woman conduct breast self-examination with
respect to her menses?
· Five to seven days following her menses
· Midcycle
· Immediately prior to menses
· During her menses
· Question Points: 1.0 / 1.0
· Jacob, a 33-year-old construction worker, complains of a
“lump on his back” over his scapula. It has been there for about
a year and is getting larger. He says his wife has been able to
squeeze out a cheesy-textured substance on occasion. He
worries this may be cancer. When gently pinched from the side,
a prominent dimple forms in the middle of the mass.
What is most likely?
· An enlarged lymph node
· A sebaceous cyst
· An actinic keratosis
· A malignant lesion
· Question Points: 1.0 / 1.0
· You have been unable to hear normal S2 splitting in children
up to this point. What technique will maximize your chances of
hearing this phenomenon?
· Listen with the diaphragm over the left lower sternal border.
· Listen with the bell over the 2nd left intercostal space.
· Listen with the bell over the apex.
· Listen with the diaphragm in the axilla.
· Question Points: 1.0 / 1.0
· A grandmother brings her 13-year-old grandson to you for
evaluation. She noticed last week when he took off his shirt that
his breastbone seemed collapsed. He seems embarrassed and
tells you that it has been that way for quite some time. He states
he has no symptoms from it, and he just tries not to take off his
shirt in front of anyone. He denies any shortness of breath,
chest pain, or lightheadedness on exertion. His past medical
history is unremarkable. He is in sixth grade and just moved in
with his grandmother after his father was deployed to the
Middle East. His mother died several years ago in a car
accident. He states that he does not smoke and has never
touched alcohol. On examination, you see a teenage boy
appearing his stated age. On visual examination, of his chest
you see that the lower portion of the sternum is depressed.
Auscultation of the lungs and heart are unremarkable.
What disorder of the thorax best describes your findings?
· Barrel chest
· Funnel chest (pectus excavatum)
· Pigeon chest (pectus carinatum)
· Thoracic kyphoscoliosis
· Question Points: 0.0 / 1.0
· Ray works a physical job and notes pain when he attempts to
lift his arm over his head. When you move the shoulder
passively, he has full range of motion without pain and there is
no gross swelling or tenderness. What type of joint disease does
this most likely represent?
· Articular
· Extra-articular
· Neither
· Both
· Question Points: 1.0 / 1.0
· Blood pressure abnormalities found more commonly in
Western elderly include which of the following?
· Isolated elevation of the diastolic BP
· Narrow pulse pressure
· Elevation of the systolic BP
· Elevation of the BP with standing
· Question Points: 1.0 / 1.0
· A 22-year-old architecture major presents to your office
complaining of severe burning with urination, a fever of 101
degrees, and aching all over. She denies any upper respiratory,
gastrointestinal, cardiac, or pulmonary symptoms. Her past
medical history consists of severe acne. She is currently on an
oral contraceptive. She has had no pregnancies or surgeries. She
reports one new partner within the last month. She does not
smoke but does drink occasionally. Her parents are both in good
health. On examination, you see a young woman appearing
slightly ill. Her temperature is 100.3 and her pulse and blood
pressure are unremarkable. Her head, ears, eyes, nose, throat,
cardiac, pulmonary, and abdominal examinations are
unremarkable. Palpation of the inguinal nodes shows
lymphadenopathy bilaterally. On visualization of the perineum,
there are more than 10 shallow ulcers along each side of the
vulva. Speculum and bimanual examination are unremarkable
for findings, although she is very tender at the introitus. Urine
analysis has some white blood cells but no red blood cells or
bacteria. Her urine pregnancy test is negative.
Which disorder of the vulva is most likely in this case?
· Genital herpes
· Condylomata acuminata
· Syphilitic chancre
· Epidermoid cyst
· Question Points: 0.0 / 1.0
· A 36-year-old married bank teller presents to your office,
complaining of pain with defecation and occasional blood on
the toilet paper. She states that last week she had food
poisoning with nausea, vomiting, and diarrhea. She had runny
stools but no black or bloody stools. Ever since her illness, she
has continued to have severe pain with bowel movements. She
now tries to put off defecation as long as possible. Although she
is having constipation she denies any further diarrhea or leakage
of stool. She has a past medical history of hypothyroidism and
two spontaneous vaginal deliveries. She has had no other
chronic illnesses or surgeries. She does not smoke and rarely
drinks. She has two children. There is no family history of
breast or colon cancer. She has had no weight gain, weight loss,
fever, or night sweats. On examination she is afebrile, with a
blood pressure of 115/70 and a pulse of 80. On abdominal
examination, she has active bowel sounds, is nontender in all
quadrants, and has no hepatosplenomegaly. Inspection of the
anus reveals inflammation on the posterior side with erythema.
Digital rectal examination is painful for the patient but no
abnormalities are palpated. Anoscopic examination reveals no
inflammation or bleeding. What is the anal disorder that best
describes her symptoms?
· Anorectal fistula
· External hemorrhoids
· Anal fissure
· Anorectal cancer
· Question Points: 0.0 / 1.0
· Adam is a very successful 15-year-old student and athlete. His
mother brings him in today because he no longer studies, works
out, or sees his friends. This has gone on for a month and a half.
When you speak with him alone in the room, he states it “would
be better if I was not here.” What would you do next?
· Tell him that he has a very promising career in anything he
chooses and soon he will feel better.
· Tell him that he needs an antidepressant and it will take about
4 weeks to work.
· Speak with his mother about getting him together more with
his friends.
· Assess his suicide risk.
· Question Points: 1.0 / 1.0
· Which of the following is a “red flag” regarding patients
presenting with headache?
· Unilateral headache
· Pain over the sinuses
· New onset over age 40
· Aggravated or relieved by change in position
· Question Points: 1.0 / 1.0
· A 73-year-old nurse presents to your office for evaluation of
new onset of pill rolling tremors. She is not on any medications
and does not take herbs or supplements. She has no chronic
medical conditions. She does not smoke or drink alcohol. She
walks into the examination room with slow movements and
shuffling steps. She has decreased facial mobility and a blunt
expression, without any changes in hair distribution on her face.
Based on this description, what is the most likely reason for the
patient's symptoms?
· Cushing's syndrome
· Nephrotic syndrome
· Myxedema
· Parkinson's disease
· Question Points: 1.0 / 1.0
· Two weeks ago, Mary started a job which requires carrying
40-pound buckets. She presents with elbow pain worse on the
right. On examination, it hurts her elbows to dorsiflex her hands
against resistance when her palms face the floor. What
condition does she have?
· Medial epicondylitis (golfer's elbow)
· Olecranon bursitis
· Lateral epicondylitis (tennis elbow)
· Supracondylar fracture
· Question Points: 0.0 / 1.0
· A 26-year-old violinist presents to your clinic complaining of
anxiety. He is a first-chair violinist in the local symphony
orchestra and has started having symptoms during
performances, such as sweating, shaking, and hyperventilating.
It has gotten so bad that he has thought about giving up his
first-chair status so he does not have to play the solo during one
of the movements. He says that he never has these symptoms
during rehearsals or when he is practicing. He denies having
any of these symptoms at any other time. His past medical
history is unremarkable. He denies any tobacco use, drug use, or
alcohol abuse. His parents are both healthy. On examination you
see a young man who appears worried. His vital signs and
physical examination are unremarkable.
What type of anxiety disorder best describes this situation?
· Panic disorder
· Specific phobia
· Social phobia
· Generalized anxiety disorder
· Question Points: 0.0 / 1.0
· A 52-year-old secretary presents to your office, complaining
about accidentally leaking urine when she coughs or sneezes.
She says this has been going on for about a year now. She
relates that she has not had a period for 2 years. She denies any
recent illness or injuries. Her past medical history is significant
for four spontaneous vaginal deliveries. She is married and has
four children. She denies alcohol, tobacco, or drug use. During
her pelvic examination, you note some atrophic vaginal tissue,
but the remainder of her pelvic, abdominal, and rectal
examinations are unremarkable.
Which type of urinary incontinence does she have?
· Stress incontinence
· Urge incontinence
· Overflow incontinence
· Mixed Incontinence
· Question Points: 0.0 / 1.0
· A 37-year-old nurse presents for evaluation of colicky right
upper quadrant abdominal pain. The pain is associated with
nausea and vomiting and occurs 1 to 2 hours after eating greasy
foods. Which one of the following physical examination
descriptions would be most consistent with the diagnosis of
cholecystitis?
· Abdomen is soft, nontender, and nondistended, without
hepatosplenomegaly or masses.
· Abdomen is soft and tender to palpation in the right lower
quadrant, without rebound or guarding.
· Abdomen is soft and tender to palpation in the right upper
quadrant with inspiration, to the point of stopping inspiration,
and there is no rebound or guarding.
· Abdomen is soft and tender to palpation in the mid-epigastric
area, without rebound or guarding.
· Question Points: 0.0 / 1.0
· A 29-year-old computer programmer presents to your office
for evaluation of a headache. The tightening sensation is located
all over the head and is of moderate intensity. It used to last
minutes, but this time it has lasted for 5 days. He denies
photophobia and nausea. He spends several hours each day at a
computer monitor/keyboard. He has tried over-the-counter
medication; it has dulled the pain but not taken it away. Based
on this description, what is your most likely diagnosis?
· Tension
· Migraine
· Cluster
· Analgesic rebound
· Question Points: 1.0 / 1.0
· An 81 year old patient complains of shortness of breath for the
past few days. On examination, you note late inspiratory
crackles in the lower third of the chest and bilateral pedal
edema that were not present a week ago. What is the most likely
explanation for these?
· Asthma
· COPD
· Bronchiectasis
· Heart failure
· Question Points: 1.0 / 1.0
· A high school soccer player “blew out his knee” when the
opposing goalie's head and shoulder struck his flexed knee
while the goalie was diving for the ball. All of the following
structures were involved in some way in his injury, but which of
the following is actually an extra-articular structure?
· Synovium
· Joint capsule
· Juxta-articular bone
· Tendons
· Question Points: 1.0 / 1.0
· A woman in her 30th week has a cervical length estimated at 1
cm. Should you be concerned?
· Yes; she may be at risk for preterm labor.
· Yes; she most likely has a bicornuate uterus.
· No; this is a normal measurement for this gestational age.
· Yes; it likely indicates the fetus is in the breech position.
· Question Points: 1.0 / 1.0
· A young woman presents in for a routine wellness
examination. You notice that her vaginal walls have deep rugae
and are slightly bluish in color. She also has a thicker white
discharge. What should you suspect?
· Hypoxia
· Varicosities
· Pregnancy
· Sexually transmitted infection
· Question Points: 1.0 / 1.0
· Ms. Whiting is a 68-year-old who presents in for her usual
follow-up visit. You notice a few flat, red, and purple lesions,
about 6 centimeters in diameter, on the ulnar aspect of her
forearms but nowhere else. She doesn't mention them. They are
tender when you examine them. What should you do?
· Conclude that these are lesions she has had for a long time.
· Wait for her to mention them before asking further questions.
· Ask how she acquired them.
· Conduct the visit as usual for the patient.
· Question Points: 1.0 / 1.0
· A 58-year-old man presents to your office complaining of
bilateral back pain that now awakens him at night. This has
been steadily increasing for the past 2 months. Which one of the
following is the most reassuring to you as a clinician in this
patient with back pain?
· Age over 50
· Pain at night
· Pain lasting more than 1 month or not responding to therapy
· Pain that is bilateral
· Question Points: 1.0 / 1.0
· A 19-year-old childcare worker presents to you for her first
prenatal visit. She cannot remember when her last period was
but thinks it was between 2 and 5 months ago. When she began
gaining weight and feeling “something” moving down there, she
did a home pregnancy test and it was positive. She states she
felt the movement about a week ago. She has had no nausea,
vomiting, fatigue, or fevers. Her past medical history is
remarkable only for irregular periods. She has been dating the
same young man for a year. She says they were not using
condoms. On examination, you see an overweight young lady
appearing her stated age. Her head, eyes, ears, nose, throat,
neck, thyroid, cardiac, …
Test 40 QS
· Question Points: 0.0 / 1.0
· Which of the following represents age-related changes in the
lungs?
· Decrease in chest wall compliance
· Speed of expiration increases
· Increase in respiratory muscle strength
· Increased elastic recoil of lung tissue
· Question Points: 0.0 / 1.0
· A 32-year-old white female presents to your clinic
complaining of overwhelming sadness. She says for the past 2
months she has had crying episodes, difficulty sleeping, and
problems with overeating. She says she used to go out with her
friends from work, but now she just wants to go home and be by
herself. She also thinks that her work productivity has been
dropping because she just is too tired to care or concentrate.
She denies any feelings of guilt or any suicidal ideation. She
states that she has never felt this way in the past. She denies
any recent illness or injuries. Her past medical history consists
of an appendectomy when she was a teenager; otherwise, she
has been healthy. She is single and works as a clerk in a
medical office. She denies tobacco, alcohol, or illegal drug use.
Her mother has high blood pressure and her father has had a
history of mental illness. On examination, you see a woman
appearing her stated age who seems quite sad. Her facial
expression does not change while you talk to her and she makes
little eye contact. She speaks so softly you cannot always
understand her. Her thought processes and content seem
unremarkable.
What type of mood disorder do you think is most likely?
· Dysthymic disorder
· Manic (bipolar) disorder
· Major depressive episode
· Anxiety disorder
· Question Points: 0.0 / 1.0
· A 40-year-old mother of two presents to your office for
consultation. She is interested in knowing what her relative
risks are for developing breast cancer. She is concerned because
her sister had unilateral breast cancer 6 years ago at age 38. The
patient reports on her history that she began having periods at
age 11 and has been fairly regular ever since, except during her
two pregnancies. Her first child arrived when she was 26 and
her second at age 28. Otherwise, she has had no health
problems. Her father has high blood pressure. Her mother had
unilateral breast cancer in her 70s. The patient denies tobacco,
alcohol, or drug use. She is a family law attorney and is
married. Her examination is essentially unremarkable.
Which risk factor of her personal and family history most puts
her in danger of getting breast cancer?
.
· Age at menarche of less than 12
· First live birth between the ages of 25 and 29
·
· First-degree relative with postmenopausal breast cancer
· First-degree relative with premenopausal breast cancer
· Question Points: 0.0 / 1.0
· Ray works a physical job and notes pain when he attempts to
lift his arm over his head. When you move the shoulder
passively, he has full range of motion without pain and there is
no gross swelling or tenderness. What type of joint disease does
this most likely represent?
· Articular
· Extra-articular
· Neither
· Both
· Question Points: 0.0 / 1.0
· A 36-year-old married bank teller presents to your office,
complaining of pain with defecation and occasional blood on
the toilet paper. She states that last week she had food
poisoning with nausea, vomiting, and diarrhea. She had runny
stools but no black or bloody stools. Ever since her illness, she
has continued to have severe pain with bowel movements. She
now tries to put off defecation as long as possible. Although she
is having constipation she denies any further diarrhea or leakage
of stool. She has a past medical history of hypothyroidism and
two spontaneous vaginal deliveries. She has had no other
chronic illnesses or surgeries. She does not smoke and rarely
drinks. She has two children. There is no family history of
breast or colon cancer. She has had no weight gain, weight loss,
fever, or night sweats. On examination she is afebrile, with a
blood pressure of 115/70 and a pulse of 80. On abdominal
examination, she has active bowel sounds, is nontender in all
quadrants, and has no hepatosplenomegaly. Inspection of the
anus reveals inflammation on the posterior side with erythema.
Digital rectal examination is painful for the patient but no
abnormalities are palpated. Anoscopic examination reveals no
inflammation or bleeding. What is the anal disorder that best
describes her symptoms?
· Anorectal fistula
· External hemorrhoids
· Anal fissure
· Anorectal cancer
· Question Points: 0.0 / 1.0
· Adam is a very successful 15-year-old student and athlete. His
mother brings him in today because he no longer studies, works
out, or sees his friends. This has gone on for a month and a half.
When you speak with him alone in the room, he states it “would
be better if I was not here.” What would you do next?
· Tell him that he has a very promising career in anything he
chooses and soon he will feel better.
· Tell him that he needs an antidepressant and it will take about
4 weeks to work.
· Speak with his mother about getting him together more with
his friends.
· Assess his suicide risk.
·
Question Points: 1.0 / 1.0
· Two weeks ago, Mary started a job which requires carrying
40-pound buckets. She presents with elbow pain worse on the
right. On examination, it hurts her elbows to dorsiflex her hands
against resistance when her palms face the floor. What
condition does she have?
· Medial epicondylitis (golfer's elbow)
· Olecranon bursitis
· Lateral epicondylitis (tennis elbow)
· Supracondylar fracture
· Question Points: 0.0 / 1.0
· A 26-year-old violinist presents to your clinic complaining of
anxiety. He is a first-chair violinist in the local symphony
orchestra and has started having symptoms during
performances, such as sweating, shaking, and hyperventilating.
It has gotten so bad that he has thought about giving up his
first-chair status so he does not have to play the solo during one
of the movements. He says that he never has these symptoms
during rehearsals or when he is practicing. He denies having
any of these symptoms at any other time. His past medical
history is unremarkable. He denies any tobacco use, drug use, or
alcohol abuse. His parents are both healthy. On examination you
see a young man who appears worried. His vital signs and
physical examination are unremarkable.
What type of anxiety disorder best describes this situation?
· Panic disorder
· Specific phobia
· Social phobia
· Generalized anxiety disorder
· Question Points: 0.0 / 1.0
· A 52-year-old secretary presents to your office, complaining
about accidentally leaking urine when she coughs or sneezes.
She says this has been going on for about a year now. She
relates that she has not had a period for 2 years. She denies any
recent illness or injuries. Her past medical history is significant
for four spontaneous vaginal deliveries. She is married and has
four children. She denies alcohol, tobacco, or drug use. During
her pelvic examination, you note some atrophic vaginal tissue,
but the remainder of her pelvic, abdominal, and rectal
examinations are unremarkable.
Which type of urinary incontinence does she have?
· Stress incontinence
· Urge incontinence
· Overflow incontinence
· Mixed Incontinence
· Question Points: 0.0 / 1.0
· A 37-year-old nurse presents for evaluation of colicky right
upper quadrant abdominal pain. The pain is associated with
nausea and vomiting and occurs 1 to 2 hours after eating greasy
foods. Which one of the following physical examination
descriptions would be most consistent with the diagnosis of
cholecystitis?
· Abdomen is soft, nontender, and nondistended, without
hepatosplenomegaly or masses.
· Abdomen is soft and tender to palpation in the right lower
quadrant, without rebound or guarding.
· Abdomen is soft and tender to palpation in the right upper
quadrant with inspiration, to the point of stopping inspiration,
and there is no rebound or guarding.
· Abdomen is soft and tender to palpation in the mid-epigastric
area, without rebound or guarding.
· Question Points: 0.0 / 1.0
· A 29-year-old computer programmer presents to your office
for evaluation of a headache. The tightening sensation is located
all over the head and is of moderate intensity. It used to last
minutes, but this time it has lasted for 5 days. He denies
photophobia and nausea. He spends several hours each day at a
computer monitor/keyboard. He has tried over-the-counter
medication; it has dulled the pain but not taken it away. Based
on this description, what is your most likely diagnosis?
· Tension
· Migraine
· Cluster
· Analgesic rebound
·
Question Points: 0.0 / 1.0
· You are having trouble examining the abdomen of a school-
aged child due to ticklishness. What should you do?
· Have the child press on your hand.
· Have the parent insist that the child allow you to examine her.
· Ask the parent to leave the room.
· Make the child realize that this is part of the examination and
must be done.
· Question Points: 1.0 / 1.0
· A 27-year-old woman is brought to your office by her mother.
The mother tells you that her daughter has been schizophrenic
for the last 8 years and is starting to decompensate despite
medication. The patient states that she has been taking her
antipsychotic and she is doing just fine. Her mother retorts that
her daughter has become quite paranoid. When asked why, the
mother gives an example about the mailman. She says that her
daughter goes and gets the mail every day and then microwaves
the letters. The patient agrees that she does this but only
because she sees the mailman flipping through the envelopes
and she knows he's putting anthrax on the letters. Her mother
turns to her and says, “He's only sorting the mail!”
Which best describes the patient's abnormality of perception?
· Illusion
· Hallucination
· Fugue state
· Facticious
· Question Points: 0.0 / 1.0
Top of Form
· A 29-year-old married computer programmer presents to your
clinic, complaining of “something strange” going on in his
scrotum. Last month while he was doing his testicular self-
examination, he felt a lump in his left testis. He waited a month
and felt the area again, but the lump was still there. He has had
some aching in his left testis but denies any pain with urination
or sexual intercourse. He denies any fever, malaise, or night
sweats. His past medical history consists of groin surgery when
he was a baby and a tonsillectomy as a teenager. He eats a
healthy diet and works out at the gym five times a week. He
denies any tobacco or illegal drugs and drinks alcohol
occasionally. His parents are both healthy. On examination, you
see a muscular, healthy, young-appearing man with
unremarkable vital signs. On visualization, the penis is
circumcised with no lesions; there is a scar in his right inguinal
region. There is no lymphadenopathy. Palpation of his scrotum
is unremarkable on the right but indicates a large mass on the
left. Placing a finger through the inguinal ring on the right, you
have the patient bear down. Nothing is felt. You attempt to
place your finger through the left inguinal ring but cannot get
above the mass. On rectal examination, his prostate is
unremarkable.
What disorder of the testes is most likely the diagnosis?
· Hydrocele
· Inguinal hernia
· Scrotal edema
· Varicocele
· Question Points: 1.0 / 1.0
Bottom of Form
· A high school football player injured his wrist in a game. He
is tender between the two tendons at the base of the thumb.
Which of the following should be considered?
· DeQuervain's tenosynovitis
· Scaphoid fracture
· Wrist sprain
· Rheumatoid arthritis
· Question Points: 0.0 / 1.0
· An adolescent male presents to your clinic with a note from
his mother stating it is okay for him to be seen today without
her presence. He has come in for his annual sports physical
required to play football. For his age his physical examination
is unremarkable, and you sign his school's physical examination
form. You decide to take this opportunity to do some health
education with him. He admits to wondering a lot lately if he is
normal. Although he is in football he really enjoys science and
computers more. He is worried that all his buddies will think he
is a geek. He is convinced he also won't get a date for the Sadie
Hawkins dance next week because the girls all think he is
boring, too. He denies any experimentation with tobacco or
alcohol, and he blushes when you mention sex. After hitting all
the pertinent age-appropriate education points, you give him his
sports physical form and he leaves.
The patient's concerns during the visit most resemble what
developmental stage of adolescence?
· Early adolescence (10 to 14 years old)
· Middle adolescence (15 to 16 years old)
· Late adolescence (17 to 20 years old)
· Adulthood
· Question Points: 0.0 / 1.0
· You are observing an infant who is able to pull to a stand,
uses “mama” and “dada” specifically, and indicates his wants by
vocalization and pointing. Where would you place this child's
developmental age?
· 12 months
· 10 months
· 8 months
· 6 months
· Question Points: 0.0 / 1.0
· A 77-year-old retired school superintendent presents to your
office, complaining of unsteady hands. He says that, for the past
6 months, when his hands are resting in his lap they shake
uncontrollably. He says when he holds them out in front of his
body the shaking diminishes, and when he uses his hands the
shaking is also better. He also complains of some difficulty
getting up out of his chair and walking around. He denies any
recent illnesses or injuries. His past medical history is
significant for high blood pressure and coronary artery disease,
requiring a stent in the past. He has been married for more than
50 years and has five children and 12 grandchildren. He denies
any tobacco, alcohol, or drug use. His mother died of a stroke in
her 70s, and his father died of a heart attack in his 60s. He has a
younger sister who has arthritis problems. His children are all
essentially healthy. On examination, you see a fine, pill-rolling
tremor of his left hand. His right shows less movement. His
cranial nerve examination is normal. He has some difficulty
rising from his chair, his gait is slow, and it takes him time to
turn around to walk back toward you. He has almost no “arm
swing” with his gait.
What type of tremor is he most likely to have?
· Resting tremor
· Postural tremor
· Intention tremor
· Drug Induced Tremor
· Question Points: 1.0 / 1.0
· A 35-year-old stockbroker presents to your office,
complaining of feeling tired and irritable. She also says she
feels like nothing ever goes her way and that nothing good ever
happens. When you ask her how long she has felt this way she
laughs and says, “Since when have I not?” She relates that she
has felt pessimistic about life in general since she was in high
school. She denies any problems with sleep, appetite, or
concentration, and states she hasn't thought about killing
herself. She reports no recent illnesses or injuries. She is single.
She smokes one pack of cigarettes a day, drinks occasionally,
and hasn't taken any illegal drugs since college. Her mother
suffers from depression and her father has high blood pressure.
On examination her vital signs and physical examination are
unremarkable.
What mental health disorder best describes her symptoms?
· Major depressive episode
· Dysthymic disorder
· Cyclothymic disorder
· Hypothryoidism
· Question Points: 0.0 / 1.0
· Alexandra is a 28-year-old editor who presents to the clinic
with abdominal pain. The pain is a dull ache, located in the
right upper quadrant, that she rates as a 3 at the least and an 8 at
the worst. The pain, which started a few weeks ago, comes and
goes, lasts for 2 to 3 hours at a time, and seems to be worse a
couple of hours after eating. She has noticed that it starts after
eating greasy foods, so she has cut down on these as much as
she can. Initially, it occurred once a week, but now it is
occurring every other day. Nothing makes it better. From this
description, which of the seven attributes of a symptom has
been omitted?
· Setting in which the symptom occurs
· Associated manifestations
· Quality
· Timing
· Question Points: 1.0 / 1.0
· Susane is a 27-year-old who has had headaches, muscle aches,
and fatigue for the last 2 months. You have completed a
thorough history, examination, and laboratory workup but have
not found a cause. What would be an appropriate next course of
action?
· A referral to a neurologist
· A referral to a rheumatologist
· To tell the patient you can't find anything
· To screen for depression
· Question Points: 0.0 / 1.0
· A 73-year-old retired accountant presents to your office for
her annual examination. She has incontinence of urine when she
coughs or sneezes. She takes several medications for control of
hypertension and diabetes. You use the DIAPPERS mnemonic to
assess the cause of her incontinence. All of the following are
items represented by the mnemonic except for:
· Atrophic vaginitis
· Depression
· Pharmaceuticals
· Restricted mobility
· Question Points: 1.0 / 1.0
· A 8-year-old patient presents to the office for evaluation of a
rash. At first, there was only one large patch, but then more
lesions erupted suddenly on the back and torso; the lesions itch.
On physical examination, you note that the pattern of eruption
is like a Christmas tree and that there are a variety of
erythematous papules and macules on the cleavage lines of the
back. Based on this description, what is the most likely
diagnosis?
· Pityriasis rosea
· Tinea versicolor
· Psoriasis
· Atopic eczema
· Question Points: 0.0 / 1.0
· A 79-year-old retired banker presents to your office for
evaluation of difficulty with urination; he gets up five to six
times per night to urinate and has to go at least that often in the
daytime. He does not feel as if his bladder empties completely;
the strength of the urinary stream is diminished. He denies
dysuria or hematuria. This problem has been present for several
years but has worsened over the last 8 months. You palpate his
prostate. What is your expected physical examination finding,
based on this description?
· Normal size, smooth
· Normal size, boggy
· Enlarged size, smooth
· Enlarged size, boggy
· Question Points: 1.0 / 1.0
· A 49-year-old administrative assistant presents to your office
for evaluation of dizziness. You elicit the information that the
dizziness is a spinning sensation of sudden onset, worse with
head position changes. The episodes last a few seconds and then
go away, and they are accompanied by intense nausea. She has
vomited one time. She denies tinnitus. You perform a physical
examination of the head and neck and note that the patient's
hearing is intact. Pt has normal results of Weber and Rinne and
that there is nystagmus. Her gait is normal. Based on this
description, what is the most likely diagnosis?
· Benign paroxysmal positional vertigo
· Vestibular neuronitis
· Ménière's disease
· Acoustic neuroma
· Question Points: 0.0 / 1.0
· An 88-year-old retired piano teacher presents for evaluation of
fatigue. You notice that her clothes are hanging loosely off her
frame and that she has lost 15 pounds. She is unaware of this.
Her husband of 63 years died a few months ago. You ask the
patient to complete a Rapid Screen for Dietary Intake. Which of
the following statements is considered to be part of this rapid
screen?
· I eat more than two meals per day.
· I drink one glass of alcohol every day.
· Without wanting to, I have lost or gained 10 pounds in the last
6 months.
· I eat with at least one other person most of the time.
·
Question Points: 1.0 / 1.0
· A 75-year-old homemaker brings her 76-year-old husband to
your clinic. She states that 4 months ago he had a stroke and
ever since she has been frustrated with his problems with
communication. They were at a restaurant after church one
Sunday when he suddenly became quiet. When she realized
something was wrong he was taken to the hospital by EMS. He
spent 2 weeks in the hospital with right-sided weakness and
difficulty speaking. After hospitalization he was in a rehab
center, where he regained the ability to walk and most of the
use of his right hand. He also began to speak more, but she says
that much of the time “he doesn't make any sense.” She gives an
example that when she reminded him the car needed to be
serviced he told her “I will change the Kool-Aid out of the sink
myself with the ludrip.” She says that these sayings are
becoming frustrating. She wants you to tell her what is wrong
and what you can do about it. While you write up a consult to
neurology, you describe the syndrome to her.
· Wernicke's aphasia
· Broca's aphasia
· Dysarthria
· Dementia
· Question Points: 0.0 / 1.0
· A 12-year-old presents to the clinic with his father for
evaluation of a painful lump at the left eyelid. It started this
morning. He denies any trauma or injury. There is no visual
disturbance. Upon physical examination, there is a red raised
area at the margin of the eyelid that is tender to palpation; no
tearing occurs with palpation of the lesion. Based on this
description, what is the most likely diagnosis?
· Dacryocystitis
· Chalazion
· Hordeolum
· Xanthelasma
· Question Points: 0.0 / 1.0
· A young man feels something in his scrotum and presents to
you for clarification. On your examination, you note what feels
like a “bag of worms” in the left scrotum, superior to the
testicles. Which of the following is most likely?
· Hydrocele of the spermatic cord
· Varicocele
· Testicular carcinoma
· A normal vas deferens
· Question Points: 0.0 / 1.0
· A 28-year-old graduate student presents to your clinic for
evaluation of pain “all over.” With further questioning, she is
able to relate that the pain is worse in the neck, shoulders,
hands, lower back, and knees. She denies swelling in her joints;
she states that the pain is worse in the morning; there is no
limitation in her range of motion. On physical examination, she
has several points on the muscles of the neck, shoulders, and
back that are tender to palpation; muscle strength and range of
motion are normal. Which of the following is likely the cause of
her pain?
· Rheumatoid arthritis
· Osteoarthritis
· Fibromyalgia
· Polymyalgia rheumatic
· Question Points: 0.0 / 1.0
· Jim is a 60-year-old man who presents with vomiting. He
denies seeing any blood with emesis, which has been occurring
for 2 days. He does note a dark, granular substance resembling
coffee grounds. What do you suspect?
· Bleeding from a diverticulum
· Bleeding from a peptic ulcer
· Bleeding from a colon cancer
· Bleeding from cholecystitis
· Question Points: 1.0 / 1.0
· The following information is recorded in the health history: “I
feel really tired.”
To which category does this information belong?
· Chief complaint
· Present illness
· Personal and social history
· Review of systems
· Question Points: 0.0 / 1.0
· A 24-year-old secretary presents to your clinic complaining of
difficulty sleeping, severe nightmares, and irritability. She
states it all began 6 months ago when she went to a fast-food
restaurant at midnight. While she was waiting in her car, a man
entered through the passenger door and put a gun to her head.
He had her drive to a remote area where he took her money and
threatened to kill her. When the gun jammed, he panicked and
ran off. Ever since this occurred, the patient has been having
these symptoms. She states she jumps at every noise and refuses
to drive at night. She states her anxiety has had such a marked
influence on her job performance that she is afraid she will be
fired. She denies any recent illnesses or injuries. Her past
medical history is unremarkable. On examination you find a
nervous woman appearing her stated age. Her physical
examination is unremarkable. You recommend medication and
counseling.
What anxiety disorder do you think this young women most
likely has?
· Specific phobia
· Acute stress disorder
· Post-traumatic stress disorder
· Generalized anxiety disorder
· Question Points: 1.0 / 1.0
· A 41-year-old real estate agent presents to your office,
complaining that he feels like his face is paralyzed on the left.
He states that last week he felt his left eyelid was drowsy and as
the day progressed he was unable to close his eyelid all the way.
Later he felt like his smile became affected also. He denies any
recent injuries but had an upper respiratory viral infection last
month. His past medical history is unremarkable. He is divorced
and has one child. He smokes one pack of cigarettes a day,
occasionally drinks alcohol, and denies any illegal drug use. His
mother has high blood pressure, and his father has sarcoidosis.
On examination, you ask him to close his eyes. He is unable to
close his left eye. You ask him to open his eyes and raise his
eyebrows. His right forehead furrows but his left remains flat.
You then ask him to give you a big smile. The right corner of
his mouth raises but the left side of his mouth remains the same.
What type of facial paralysis does he have?
· Peripheral CN VII paralysis
· Central CN VII paralysis
· CN V Neuralgia
· CN V paralysis
· Question Points: 0.0 / 1.0
· Which of the following booster immunizations is
recommended in the older adult population?
· Tetanus
· Diphtheria
· Measles
· Mumps
· Question Points: 0.0 / 1.0
· A 26-year-old white female presents to your clinic at 38
weeks, complaining of intermittent contractions. They last for
30 seconds and are coming every 10 minutes. Her prenatal
course has so far been uneventful. You send her to labor and
delivery for a labor assessment. On vaginal examination, she
has effaced 4 cm, but you cannot feel a presenting part. You
admit her for active labor; however, you wish to assess if she is
vertex (baby's head is down), so you do the Leopold's
maneuver. Palpating the upper pole with your hands, you feel a
firm round mass. Placing your hand along the right side of her
abdomen, you feel a smooth firmness. Palpating your other hand
along the left side of her abdomen, you feel irregular bumps.
Above the pelvic brim you feel a firm irregular mass. While
awaiting ultrasound to confirm your diagnosis, you write the
pertinent orders.
How is this fetus presenting?
· Vertex
· Breech
· Transverse
· Anteverted
· Question Points: 1.0 / 1.0
· A 29-year-old homemaker who is G4P3 presents to your clinic
for her first prenatal check. Her last period was 2 months ago.
She has had three previous pregnancies and deliveries with no
complications. She has no medical problems and has had no
surgeries. Her only current complaint is of severe reflux that
occurs in the mornings and evenings. On examination she is in
no acute distress. Her vitals are 110/70 with a pulse of 88. Her
respirations are 16. Her head, eyes, ears, nose, throat, thyroid,
cardiac, pulmonary, and abdominal examinations are
unremarkable. On bimanual examination her cervix is soft and
her uterus is 10 weeks in size. Pap smear, cultures, and blood
work are pending.
What is the most likely cause of her first-trimester reflux?
· Increasing prolactin levels
· Increasing ADH (antidiuretic hormone) levels
· Increasing progesterone
· Enlarged gravid uterus
· Question Points: 0.0 / 1.0
· A patient is assigned a visual acuity of 20/100 in her left eye.
Which of the following is true?
· She obtains a 20% correct score at 100 feet.
· She can accurately name 20% of the letters at 20 feet.
· She can see at 20 feet what a normal person could see at 100
feet.
· She can see at 100 feet what a normal person could see at 20
feet.
· Question Points: 1.0 / 1.0
· A 72-year-old African-American male is brought to your clinic
by his daughter for a follow-up visit after his recent
hospitalization. He had been admitted to the local hospital for
speech …
NUR-634 Final Exam Test Prep· You are examining an elderly man.docx

More Related Content

Similar to NUR-634 Final Exam Test Prep· You are examining an elderly man.docx

Breast Cancer SOAP noteName Sharon BroomDate Januar
Breast Cancer SOAP  noteName  Sharon BroomDate JanuarBreast Cancer SOAP  noteName  Sharon BroomDate Januar
Breast Cancer SOAP noteName Sharon BroomDate Januar
CicelyBourqueju
 
MIDTERM 634Is the following information subjective or objective.docx
MIDTERM  634Is the following information subjective or objective.docxMIDTERM  634Is the following information subjective or objective.docx
MIDTERM 634Is the following information subjective or objective.docx
endawalling
 
SOAP NOTEName J.D.Date 03262020Time 200 pmAge 25 .docx
SOAP NOTEName J.D.Date 03262020Time 200 pmAge 25 .docxSOAP NOTEName J.D.Date 03262020Time 200 pmAge 25 .docx
SOAP NOTEName J.D.Date 03262020Time 200 pmAge 25 .docx
pbilly1
 
Name  Anna Jones  Pt. Encounter Numb
Name  Anna Jones                         Pt. Encounter NumbName  Anna Jones                         Pt. Encounter Numb
Name  Anna Jones  Pt. Encounter Numb
TaunyaCoffman887
 
R2 management of menstrual disordersll
R2 management of menstrual disordersllR2 management of menstrual disordersll
R2 management of menstrual disordersll
Amir Mahmoud
 
N544- Physical Assessment of the Male GU System, Anus, Rectum and Prostate- u...
N544- Physical Assessment of the Male GU System, Anus, Rectum and Prostate- u...N544- Physical Assessment of the Male GU System, Anus, Rectum and Prostate- u...
N544- Physical Assessment of the Male GU System, Anus, Rectum and Prostate- u...
MulugetaAbeneh1
 
WU SOAP Analysis Discussion Reply.docx
WU SOAP Analysis Discussion Reply.docxWU SOAP Analysis Discussion Reply.docx
WU SOAP Analysis Discussion Reply.docx
write5
 

Similar to NUR-634 Final Exam Test Prep· You are examining an elderly man.docx (20)

Contraception Essays.docx
Contraception Essays.docxContraception Essays.docx
Contraception Essays.docx
 
Breast Cancer SOAP noteName Sharon BroomDate Januar
Breast Cancer SOAP  noteName  Sharon BroomDate JanuarBreast Cancer SOAP  noteName  Sharon BroomDate Januar
Breast Cancer SOAP noteName Sharon BroomDate Januar
 
Ca rectum presentation.pptx
Ca rectum presentation.pptxCa rectum presentation.pptx
Ca rectum presentation.pptx
 
UWSA 1 2021 For USMLE Step 1 Exam
UWSA 1 2021 For USMLE Step 1 ExamUWSA 1 2021 For USMLE Step 1 Exam
UWSA 1 2021 For USMLE Step 1 Exam
 
OSCE REVISION .pptx
OSCE REVISION .pptxOSCE REVISION .pptx
OSCE REVISION .pptx
 
[MBBS/MS/DNB] Sample EXAM Long Case on Breast Lump
[MBBS/MS/DNB] Sample EXAM Long Case on Breast Lump [MBBS/MS/DNB] Sample EXAM Long Case on Breast Lump
[MBBS/MS/DNB] Sample EXAM Long Case on Breast Lump
 
MIDTERM 634Is the following information subjective or objective.docx
MIDTERM  634Is the following information subjective or objective.docxMIDTERM  634Is the following information subjective or objective.docx
MIDTERM 634Is the following information subjective or objective.docx
 
Gyn into....
Gyn into....Gyn into....
Gyn into....
 
SOAP NOTEName J.D.Date 03262020Time 200 pmAge 25 .docx
SOAP NOTEName J.D.Date 03262020Time 200 pmAge 25 .docxSOAP NOTEName J.D.Date 03262020Time 200 pmAge 25 .docx
SOAP NOTEName J.D.Date 03262020Time 200 pmAge 25 .docx
 
Case Presentation - Final Edit - Copy.pptx
Case Presentation - Final Edit - Copy.pptxCase Presentation - Final Edit - Copy.pptx
Case Presentation - Final Edit - Copy.pptx
 
Name  Anna Jones  Pt. Encounter Numb
Name  Anna Jones                         Pt. Encounter NumbName  Anna Jones                         Pt. Encounter Numb
Name  Anna Jones  Pt. Encounter Numb
 
Acute Leukemia
Acute LeukemiaAcute Leukemia
Acute Leukemia
 
R2 management of menstrual disordersll
R2 management of menstrual disordersllR2 management of menstrual disordersll
R2 management of menstrual disordersll
 
Surgery case presentation: A 50 year old lady with a lump in the right breast
Surgery case presentation: A 50 year old lady with a lump in the right breastSurgery case presentation: A 50 year old lady with a lump in the right breast
Surgery case presentation: A 50 year old lady with a lump in the right breast
 
Case presentation gastrology
Case presentation gastrologyCase presentation gastrology
Case presentation gastrology
 
N544- Physical Assessment of the Male GU System, Anus, Rectum and Prostate- u...
N544- Physical Assessment of the Male GU System, Anus, Rectum and Prostate- u...N544- Physical Assessment of the Male GU System, Anus, Rectum and Prostate- u...
N544- Physical Assessment of the Male GU System, Anus, Rectum and Prostate- u...
 
WU SOAP Analysis Discussion Reply.docx
WU SOAP Analysis Discussion Reply.docxWU SOAP Analysis Discussion Reply.docx
WU SOAP Analysis Discussion Reply.docx
 
A case of testicular Malignancy
A case of testicular MalignancyA case of testicular Malignancy
A case of testicular Malignancy
 
Carcinoma Breast.pptx
Carcinoma Breast.pptxCarcinoma Breast.pptx
Carcinoma Breast.pptx
 
Case Write Up Surgical Gastric Carcinoma
Case Write Up Surgical Gastric CarcinomaCase Write Up Surgical Gastric Carcinoma
Case Write Up Surgical Gastric Carcinoma
 

More from dunhamadell

One of the most important ways we can strengthen our relationships w.docx
One of the most important ways we can strengthen our relationships w.docxOne of the most important ways we can strengthen our relationships w.docx
One of the most important ways we can strengthen our relationships w.docx
dunhamadell
 
one paragraph with intext citation and reference follow up discussio.docx
one paragraph with intext citation and reference follow up discussio.docxone paragraph with intext citation and reference follow up discussio.docx
one paragraph with intext citation and reference follow up discussio.docx
dunhamadell
 
One of the main jobs of historians is to interpret the past by revie.docx
One of the main jobs of historians is to interpret the past by revie.docxOne of the main jobs of historians is to interpret the past by revie.docx
One of the main jobs of historians is to interpret the past by revie.docx
dunhamadell
 
One of the examples I chose for my readings was, For Coloured .docx
One of the examples I chose for my readings was,  For Coloured .docxOne of the examples I chose for my readings was,  For Coloured .docx
One of the examples I chose for my readings was, For Coloured .docx
dunhamadell
 
One of the many reasons social workers conduct  needs assessment.docx
One of the many reasons social workers conduct  needs assessment.docxOne of the many reasons social workers conduct  needs assessment.docx
One of the many reasons social workers conduct  needs assessment.docx
dunhamadell
 
one 2-3 page essay (typed and double-spaced). Your essay should have.docx
one 2-3 page essay (typed and double-spaced). Your essay should have.docxone 2-3 page essay (typed and double-spaced). Your essay should have.docx
one 2-3 page essay (typed and double-spaced). Your essay should have.docx
dunhamadell
 
One implication of diverse cultures is that elements of one cult.docx
One implication of diverse cultures is that elements of one cult.docxOne implication of diverse cultures is that elements of one cult.docx
One implication of diverse cultures is that elements of one cult.docx
dunhamadell
 
On the tomcat drive in folder cosc210 you will find file named Paint.docx
On the tomcat drive in folder cosc210 you will find file named Paint.docxOn the tomcat drive in folder cosc210 you will find file named Paint.docx
On the tomcat drive in folder cosc210 you will find file named Paint.docx
dunhamadell
 

More from dunhamadell (20)

One of the most important ways we can strengthen our relationships w.docx
One of the most important ways we can strengthen our relationships w.docxOne of the most important ways we can strengthen our relationships w.docx
One of the most important ways we can strengthen our relationships w.docx
 
One of the five elements of emotional intelligence is self-awareness.docx
One of the five elements of emotional intelligence is self-awareness.docxOne of the five elements of emotional intelligence is self-awareness.docx
One of the five elements of emotional intelligence is self-awareness.docx
 
One of the challenges facing the Public Information Officer (PIO.docx
One of the challenges facing the Public Information Officer (PIO.docxOne of the challenges facing the Public Information Officer (PIO.docx
One of the challenges facing the Public Information Officer (PIO.docx
 
One popular measure of human capital today is employee engagement, w.docx
One popular measure of human capital today is employee engagement, w.docxOne popular measure of human capital today is employee engagement, w.docx
One popular measure of human capital today is employee engagement, w.docx
 
one paragraph with intext citation and reference follow up discussio.docx
one paragraph with intext citation and reference follow up discussio.docxone paragraph with intext citation and reference follow up discussio.docx
one paragraph with intext citation and reference follow up discussio.docx
 
One page please!!1. Using conflict perspective, discuss the ca.docx
One page please!!1. Using conflict perspective, discuss the ca.docxOne page please!!1. Using conflict perspective, discuss the ca.docx
One page please!!1. Using conflict perspective, discuss the ca.docx
 
One page per question What is a political party How do the tw.docx
One page per question What is a political party How do the tw.docxOne page per question What is a political party How do the tw.docx
One page per question What is a political party How do the tw.docx
 
One of the biggest challenges in serial killer investigation has.docx
One of the biggest challenges in serial killer investigation has.docxOne of the biggest challenges in serial killer investigation has.docx
One of the biggest challenges in serial killer investigation has.docx
 
One of the main jobs of historians is to interpret the past by revie.docx
One of the main jobs of historians is to interpret the past by revie.docxOne of the main jobs of historians is to interpret the past by revie.docx
One of the main jobs of historians is to interpret the past by revie.docx
 
One of the responsibilities of the medical administrator is hand.docx
One of the responsibilities of the medical administrator is hand.docxOne of the responsibilities of the medical administrator is hand.docx
One of the responsibilities of the medical administrator is hand.docx
 
One of the examples I chose for my readings was, For Coloured .docx
One of the examples I chose for my readings was,  For Coloured .docxOne of the examples I chose for my readings was,  For Coloured .docx
One of the examples I chose for my readings was, For Coloured .docx
 
One of the many reasons social workers conduct  needs assessment.docx
One of the many reasons social workers conduct  needs assessment.docxOne of the many reasons social workers conduct  needs assessment.docx
One of the many reasons social workers conduct  needs assessment.docx
 
one 2-3 page essay (typed and double-spaced). Your essay should have.docx
one 2-3 page essay (typed and double-spaced). Your essay should have.docxone 2-3 page essay (typed and double-spaced). Your essay should have.docx
one 2-3 page essay (typed and double-spaced). Your essay should have.docx
 
One complex issue that a public organization has to monitor on a.docx
One complex issue that a public organization has to monitor on a.docxOne complex issue that a public organization has to monitor on a.docx
One complex issue that a public organization has to monitor on a.docx
 
One implication of diverse cultures is that elements of one cult.docx
One implication of diverse cultures is that elements of one cult.docxOne implication of diverse cultures is that elements of one cult.docx
One implication of diverse cultures is that elements of one cult.docx
 
On the project topic of Phishing. Please complete the below tasks.docx
On the project topic of Phishing. Please complete the below tasks.docxOn the project topic of Phishing. Please complete the below tasks.docx
On the project topic of Phishing. Please complete the below tasks.docx
 
On the tomcat drive in folder cosc210 you will find file named Paint.docx
On the tomcat drive in folder cosc210 you will find file named Paint.docxOn the tomcat drive in folder cosc210 you will find file named Paint.docx
On the tomcat drive in folder cosc210 you will find file named Paint.docx
 
On November 3, 2020, Californias Privacy Rights Act is on the b.docx
On November 3, 2020, Californias Privacy Rights Act is on the b.docxOn November 3, 2020, Californias Privacy Rights Act is on the b.docx
On November 3, 2020, Californias Privacy Rights Act is on the b.docx
 
On Human Capital (250 Words)The neoliberal understanding of h.docx
On Human Capital (250 Words)The neoliberal understanding of h.docxOn Human Capital (250 Words)The neoliberal understanding of h.docx
On Human Capital (250 Words)The neoliberal understanding of h.docx
 
ompile the information you have gathered on your companies in weeks .docx
ompile the information you have gathered on your companies in weeks .docxompile the information you have gathered on your companies in weeks .docx
ompile the information you have gathered on your companies in weeks .docx
 

Recently uploaded

Spellings Wk 3 English CAPS CARES Please Practise
Spellings Wk 3 English CAPS CARES Please PractiseSpellings Wk 3 English CAPS CARES Please Practise
Spellings Wk 3 English CAPS CARES Please Practise
AnaAcapella
 

Recently uploaded (20)

Food safety_Challenges food safety laboratories_.pdf
Food safety_Challenges food safety laboratories_.pdfFood safety_Challenges food safety laboratories_.pdf
Food safety_Challenges food safety laboratories_.pdf
 
ICT role in 21st century education and it's challenges.
ICT role in 21st century education and it's challenges.ICT role in 21st century education and it's challenges.
ICT role in 21st century education and it's challenges.
 
Mehran University Newsletter Vol-X, Issue-I, 2024
Mehran University Newsletter Vol-X, Issue-I, 2024Mehran University Newsletter Vol-X, Issue-I, 2024
Mehran University Newsletter Vol-X, Issue-I, 2024
 
2024-NATIONAL-LEARNING-CAMP-AND-OTHER.pptx
2024-NATIONAL-LEARNING-CAMP-AND-OTHER.pptx2024-NATIONAL-LEARNING-CAMP-AND-OTHER.pptx
2024-NATIONAL-LEARNING-CAMP-AND-OTHER.pptx
 
Unit-IV- Pharma. Marketing Channels.pptx
Unit-IV- Pharma. Marketing Channels.pptxUnit-IV- Pharma. Marketing Channels.pptx
Unit-IV- Pharma. Marketing Channels.pptx
 
Python Notes for mca i year students osmania university.docx
Python Notes for mca i year students osmania university.docxPython Notes for mca i year students osmania university.docx
Python Notes for mca i year students osmania university.docx
 
ICT Role in 21st Century Education & its Challenges.pptx
ICT Role in 21st Century Education & its Challenges.pptxICT Role in 21st Century Education & its Challenges.pptx
ICT Role in 21st Century Education & its Challenges.pptx
 
UGC NET Paper 1 Mathematical Reasoning & Aptitude.pdf
UGC NET Paper 1 Mathematical Reasoning & Aptitude.pdfUGC NET Paper 1 Mathematical Reasoning & Aptitude.pdf
UGC NET Paper 1 Mathematical Reasoning & Aptitude.pdf
 
Accessible Digital Futures project (20/03/2024)
Accessible Digital Futures project (20/03/2024)Accessible Digital Futures project (20/03/2024)
Accessible Digital Futures project (20/03/2024)
 
Sociology 101 Demonstration of Learning Exhibit
Sociology 101 Demonstration of Learning ExhibitSociology 101 Demonstration of Learning Exhibit
Sociology 101 Demonstration of Learning Exhibit
 
How to Give a Domain for a Field in Odoo 17
How to Give a Domain for a Field in Odoo 17How to Give a Domain for a Field in Odoo 17
How to Give a Domain for a Field in Odoo 17
 
Spatium Project Simulation student brief
Spatium Project Simulation student briefSpatium Project Simulation student brief
Spatium Project Simulation student brief
 
Kodo Millet PPT made by Ghanshyam bairwa college of Agriculture kumher bhara...
Kodo Millet  PPT made by Ghanshyam bairwa college of Agriculture kumher bhara...Kodo Millet  PPT made by Ghanshyam bairwa college of Agriculture kumher bhara...
Kodo Millet PPT made by Ghanshyam bairwa college of Agriculture kumher bhara...
 
Spellings Wk 3 English CAPS CARES Please Practise
Spellings Wk 3 English CAPS CARES Please PractiseSpellings Wk 3 English CAPS CARES Please Practise
Spellings Wk 3 English CAPS CARES Please Practise
 
Making communications land - Are they received and understood as intended? we...
Making communications land - Are they received and understood as intended? we...Making communications land - Are they received and understood as intended? we...
Making communications land - Are they received and understood as intended? we...
 
Third Battle of Panipat detailed notes.pptx
Third Battle of Panipat detailed notes.pptxThird Battle of Panipat detailed notes.pptx
Third Battle of Panipat detailed notes.pptx
 
psychiatric nursing HISTORY COLLECTION .docx
psychiatric  nursing HISTORY  COLLECTION  .docxpsychiatric  nursing HISTORY  COLLECTION  .docx
psychiatric nursing HISTORY COLLECTION .docx
 
Micro-Scholarship, What it is, How can it help me.pdf
Micro-Scholarship, What it is, How can it help me.pdfMicro-Scholarship, What it is, How can it help me.pdf
Micro-Scholarship, What it is, How can it help me.pdf
 
Application orientated numerical on hev.ppt
Application orientated numerical on hev.pptApplication orientated numerical on hev.ppt
Application orientated numerical on hev.ppt
 
SOC 101 Demonstration of Learning Presentation
SOC 101 Demonstration of Learning PresentationSOC 101 Demonstration of Learning Presentation
SOC 101 Demonstration of Learning Presentation
 

NUR-634 Final Exam Test Prep· You are examining an elderly man.docx

  • 1. NUR-634 Final Exam Test Prep · You are examining an elderly man and notice the following: Decreased vibration sense in the feet and ankles, diminished gag reflex, right patellar reflex less than the left, and diminished abdominal reflexes. Which of these is abnormal? · Decreased vibration sense · Diminished gag reflex · Diminished right patellar reflex compared to the left · Diminished abdominal reflexes · Question Points: 1.0 / 1.0 · A 68-year-old retired farmer presents to your office for evaluation of a skin lesion. On the right temporal area of the forehead, you see a flattened papule the same color as his skin, covered by a dry scale that is round and feels hard. He has several more of these scattered on the forehead, arms, and legs. Based on this description, what is your most likely diagnosis? · Actinic keratosis · Seborrheic keratosis · Basal cell carcinoma · Squamous cell carcinoma · Question Points: 1.0 / 1.0
  • 2. · A 14-year-old junior high school student is brought in by his mother and father because he seems to be developing breasts. The mother is upset because she read on the Internet that smoking marijuana leads to breast enlargement in males. The young man adamantly denies using any tobacco, alcohol, or drugs. He has recently noticed changes in his penis, testicles, and pubic hair pattern. Otherwise, his past medical history is unremarkable. His parents are both in good health. He has two older brothers who never had this problem. On examination, you see a mildly overweight teenager with enlarged breast tissue that is slightly tender on both sides. Otherwise his examination is normal. He is agreeable to taking a drug test. What is the most likely cause of his gynecomastia? · Breast cancer · Imbalance of hormones of puberty · Drug use · Alcohol use · Question Points: 1.0 / 1.0 · A 17-year-old female presents to your office, complaining of a clear discharge from her right breast for 2 months. She states that she noticed it when she and her boyfriend were “messing around” and he squeezed her nipple. She continues to have this discharge anytime she squeezes that nipple. She denies any trauma to her breasts. Her past medical history is unremarkable. She denies any pregnancies. Both of her parents are healthy. She denies using tobacco or illegal drugs and drinks three to four beers a week. On examination, her breasts are symmetric with no skin changes. You are able to express clear discharge from her right nipple. You feel no discrete masses and her axillae are normal. The remainder of her heart, lung, abdominal, and pelvic examinations are unremarkable. A urine pregnancy
  • 3. test is negative. What cause of nipple discharge is the most likely in her circumstance? · Benign breast abnormality · Breast cancer · Nonpuerperal galactorrhea · Pagets disease · Question Points: 1.0 / 1.0 · A 26-year-old flight attendant presents in for a third trimester prenatal visit. She has had prenatal care since her sixth week of pregnancy. She has no complaints today and her prenatal course has been unremarkable. Today her blood pressure and weight gain are appropriate, and her urine is unremarkable. You have a first-year medical student shadowing you, so you ask the student to get Doptones and measure the patient's uterus in centimeters. The nurse practitioner student promptly reports fetal heart tones of 140, but he is having difficulty obtaining the correct measurement. He knows one end of the tape goes over the uterine fundus. From what inferior anatomic position should the tape be placed? · Vagina · Clitoris · Pubic symphysis · Umbilicus · Question Points: 1.0 / 1.0
  • 4. · A 58-year-old gardener presents to your office for evaluation of a new lesion on her upper chest. The lesion appears to be “stuck on” and is oval, brown, and slightly elevated with a flat surface. It has a rough, wart like texture on palpation. Based on this description, what is your most likely diagnosis? · Actinic keratosis · Seborrheic keratosis · Basal cell carcinoma · Squamous cell carcinoma · Question Points: 0.0 / 1.0 · Which of the following represents age-related changes in the lungs? · Decrease in chest wall compliance · Speed of expiration increases · Increase in respiratory muscle strength · Increased elastic recoil of lung tissue · Question Points: 1.0 / 1.0 Top of Form · A 15-month-old is brought to you for a fever of 38.6 degrees Celsius and fussiness. The ear examination is as follows: external ear, normal appearance and no tenderness with manipulation; canal, normal diameter without evidence of inflammation; tympanic membrane, bulging, erythematous, and
  • 5. opaque. Insufflation is deferred due to pain. What is the most likely condition here? · Otitis externa · Cholesteatoma · Ruptured tympanic membrane · Otitis media · Question Points: 1.0 / 1.0 Bottom of Form · A 28-year-old musician presents to your clinic, complaining of a “spot” on his penis. He states his partner noticed it 2 days ago and it hasn't gone away. He says it doesn't hurt. He has had no burning with urination and no pain during intercourse. He has had several partners in the last year and uses condoms occasionally. His past medical history consists of nongonococcal urethritis from Chlamydia and prostatitis. He denies any surgeries. He smokes two packs of cigarettes a day, drinks a case of beer a week, and smokes marijuana and occasionally crack. He has injected IV drugs before but not in the last few years. He is single and currently unemployed. His mother has rheumatoid arthritis, and he doesn't know anything about his father. On examination, you see a young man appearing deconditioned but pleasant. His vital signs are unremarkable. On visualization of his penis, there is a 6-mm red, oval ulcer with an indurated base just proximal to the corona. There is no prepuce because of neonatal circumcision. On palpation, the ulcer is nontender. In the inguinal region, there is nontender lymphadenopathy. What disorder of the penis is most likely the diagnosis? · Condylomata acuminata
  • 6. · Genital herpes · Syphilitic chancre · Penile carcinoma · Question Points: 1.0 / 1.0 · A 22-year-old law student presents to your office complaining of severe abdominal pain radiating to his back. He states it began last night after hours of heavy drinking. He has had abdominal pain and vomiting in the past after drinking but never as bad as this. He cannot keep any food or water down, and these symptoms have been going on for almost 12 hours. He has had no recent illnesses or injuries. His past medical history is unremarkable. He denies smoking or using illegal drugs but admits to drinking 6 to 10 beers per weekend night. He admits that last night he drank something like 14 drinks. On examination you find a young male appearing his stated age in some distress. He is leaning over on the examination table and holding his abdomen with his arms. His blood pressure is 90/60 and his pulse is 120. He is afebrile. His abdominal examination reveals normal bowel sounds, but he is very tender in the left upper quadrant and epigastric area. He has no Murphy's sign or tenderness in the right lower quadrant. The remainder of his abdominal examination is normal. His rectal, prostate, penile, and testicular examinations are normal. He has no inguinal hernias or tenderness with that examination. Blood work is pending. What etiology of abdominal pain is most likely causing his symptoms? · Peptic ulcer disease · Biliary colic
  • 7. · Acute cholecystitis · Acute pancreatitis · Question Points: 1.0 / 1.0 · A 56-year-old female presents to your clinic complaining that her left breast looks unusual. She says that for 2 months the angle of the nipple has changed direction. She does not do self- examinations, so she doesn't know if she has a lump. She has no history of weight loss, weight gain, fever, or night sweats. Her past medical history is significant for high blood pressure. She smokes two packs of cigarettes a day and has three to four drinks per weekend night. Her paternal aunt died of breast cancer in her forties. Her mother is healthy, but her father died of prostate cancer. On examination, you find a middle-aged woman appearing older than her stated age. Inspection of her left breast reveals a flattened nipple deviating toward the lateral side. On palpation, the nipple feels thickened. Lateral to the areola you palpate a nontender 4-cm mass. The axilla contains several fixed nodes. The right breast and axilla examinations are unremarkable. What visible skin change of the breast does she have? · Nipple retraction · Paget's disease · Peau d'orange sign · Mastitis · Question Points: 0.0 / 1.0
  • 8. · A 32-year-old white female presents to your clinic complaining of overwhelming sadness. She says for the past 2 months she has had crying episodes, difficulty sleeping, and problems with overeating. She says she used to go out with her friends from work, but now she just wants to go home and be by herself. She also thinks that her work productivity has been dropping because she just is too tired to care or concentrate. She denies any feelings of guilt or any suicidal ideation. She states that she has never felt this way in the past. She denies any recent illness or injuries. Her past medical history consists of an appendectomy when she was a teenager; otherwise, she has been healthy. She is single and works as a clerk in a medical office. She denies tobacco, alcohol, or illegal drug use. Her mother has high blood pressure and her father has had a history of mental illness. On examination, you see a woman appearing her stated age who seems quite sad. Her facial expression does not change while you talk to her and she makes little eye contact. She speaks so softly you cannot always understand her. Her thought processes and content seem unremarkable. What type of mood disorder do you think is most likely? · Dysthymic disorder · Manic (bipolar) disorder · Major depressive episode · Anxiety disorder · Question Points: 1.0 / 1.0 · A young Hispanic mother brings in her 2-month-old son. She is upset because her neighbors have threatened to call the Child Protective Agency because they think his birthmark is a bruise. Her son was the product of an uneventful pregnancy and
  • 9. spontaneous vaginal delivery. On examination, you see a large, smooth-bordered bluish mark on his buttock and lower back. Otherwise his examination is unremarkable. What form of birthmark is this likely to be? · Café-au-lait spot · Salmon patch · Mongolian spot · Cherry hemangioma · Question Points: 1.0 / 1.0 · A 15-year-old high school sophomore presents to the clinic for evaluation of a 3-week history of sneezing; itchy, watery eyes; clear nasal discharge; ear pain; and nonproductive cough. Which is the most likely pathologic process? · Infection · Inflammation · Allergic · Vascular · Question Points: 1.0 / 1.0 · A 22-year-old clerk, primigravida, presents to your office for a prenatal visit. She is in her second trimester and has had prenatal care since she was 8 weeks pregnant. Her only complaint is that she has a new brownish line straight down her abdomen. On examination her vital signs are unremarkable. Her urine has no protein, glucose, or leukocytes. With a doppler the
  • 10. fetal heart rate is 140, and her uterus is palpated to the umbilicus. Today you are sending her for congenital abnormality screening and setting up an ultrasound. What physical finding is responsible for her new “brown line”? · Corpus luteum · Linea nigra · Linea alba · Diastasis recti · Question Points: 1.0 / 1.0 · Mr. Q. is a 45-year-old salesman who presents to your office for evaluation of fatigue. He has come to the office many times in the past with a variety of injuries, and you suspect that he has a problem with alcohol. Which one of the following questions will be most helpful in diagnosing this problem? · You are an alcoholic, aren't you? · When was your last drink? · Do you drink 2 to 3 beers every weekend? · Do you drink alcohol when you are supposed to be working? · Question Points: 1.0 / 1.0 · A 35-year-old bus driver presents to your office for a prenatal visit. She is approximately 28 weeks pregnant and has had no complications. She is complaining only of heartburn and has had no fatigue, headaches, leg swelling, contractions, leakage of fluid, or bleeding. On examination, her blood pressure is 142/92
  • 11. and her urine shows no glucose, protein, or leukocytes. Her weight gain is appropriate, with no large recent increases. Fetal tones are 140 and her uterus measures 32 cm from the pubic bone. Looking back through her chart, you see her prenatal blood pressure was 120/70 and her blood pressures during the first 20 weeks were usually 120 to 130/70 to 80. What type of blood pressure is this? · Normotensive for pregnancy · Chronic hypertension · Gestational hypertension · Preeclampsia · Question Points: 1.0 / 1.0 · A sudden, painless unilateral vision loss may be caused by which of the following? · Retinal detachment · Corneal ulcer · Acute glaucoma · Uveitis · Question Points: 0.0 / 1.0 · A 40-year-old mother of two presents to your office for consultation. She is interested in knowing what her relative risks are for developing breast cancer. She is concerned because her sister had unilateral breast cancer 6 years ago at age 38. The patient reports on her history that she began having periods at
  • 12. age 11 and has been fairly regular ever since, except during her two pregnancies. Her first child arrived when she was 26 and her second at age 28. Otherwise, she has had no health problems. Her father has high blood pressure. Her mother had unilateral breast cancer in her 70s. The patient denies tobacco, alcohol, or drug use. She is a family law attorney and is married. Her examination is essentially unremarkable. Which risk factor of her personal and family history most puts her in danger of getting breast cancer? · First-degree relative with premenopausal breast cancer · Age at menarche of less than 12 · First live birth between the ages of 25 and 29 · First-degree relative with postmenopausal breast cancer · Question Points: 1.0 / 1.0 · When should a woman conduct breast self-examination with respect to her menses? · Five to seven days following her menses · Midcycle · Immediately prior to menses · During her menses · Question Points: 1.0 / 1.0 · Jacob, a 33-year-old construction worker, complains of a “lump on his back” over his scapula. It has been there for about a year and is getting larger. He says his wife has been able to
  • 13. squeeze out a cheesy-textured substance on occasion. He worries this may be cancer. When gently pinched from the side, a prominent dimple forms in the middle of the mass. What is most likely? · An enlarged lymph node · A sebaceous cyst · An actinic keratosis · A malignant lesion · Question Points: 1.0 / 1.0 · You have been unable to hear normal S2 splitting in children up to this point. What technique will maximize your chances of hearing this phenomenon? · Listen with the diaphragm over the left lower sternal border. · Listen with the bell over the 2nd left intercostal space. · Listen with the bell over the apex. · Listen with the diaphragm in the axilla. · Question Points: 1.0 / 1.0 · A grandmother brings her 13-year-old grandson to you for evaluation. She noticed last week when he took off his shirt that his breastbone seemed collapsed. He seems embarrassed and tells you that it has been that way for quite some time. He states he has no symptoms from it, and he just tries not to take off his shirt in front of anyone. He denies any shortness of breath, chest pain, or lightheadedness on exertion. His past medical
  • 14. history is unremarkable. He is in sixth grade and just moved in with his grandmother after his father was deployed to the Middle East. His mother died several years ago in a car accident. He states that he does not smoke and has never touched alcohol. On examination, you see a teenage boy appearing his stated age. On visual examination, of his chest you see that the lower portion of the sternum is depressed. Auscultation of the lungs and heart are unremarkable. What disorder of the thorax best describes your findings? · Barrel chest · Funnel chest (pectus excavatum) · Pigeon chest (pectus carinatum) · Thoracic kyphoscoliosis · Question Points: 0.0 / 1.0 · Ray works a physical job and notes pain when he attempts to lift his arm over his head. When you move the shoulder passively, he has full range of motion without pain and there is no gross swelling or tenderness. What type of joint disease does this most likely represent? · Articular · Extra-articular · Neither · Both · Question Points: 1.0 / 1.0
  • 15. · Blood pressure abnormalities found more commonly in Western elderly include which of the following? · Isolated elevation of the diastolic BP · Narrow pulse pressure · Elevation of the systolic BP · Elevation of the BP with standing · Question Points: 1.0 / 1.0 · A 22-year-old architecture major presents to your office complaining of severe burning with urination, a fever of 101 degrees, and aching all over. She denies any upper respiratory, gastrointestinal, cardiac, or pulmonary symptoms. Her past medical history consists of severe acne. She is currently on an oral contraceptive. She has had no pregnancies or surgeries. She reports one new partner within the last month. She does not smoke but does drink occasionally. Her parents are both in good health. On examination, you see a young woman appearing slightly ill. Her temperature is 100.3 and her pulse and blood pressure are unremarkable. Her head, ears, eyes, nose, throat, cardiac, pulmonary, and abdominal examinations are unremarkable. Palpation of the inguinal nodes shows lymphadenopathy bilaterally. On visualization of the perineum, there are more than 10 shallow ulcers along each side of the vulva. Speculum and bimanual examination are unremarkable for findings, although she is very tender at the introitus. Urine analysis has some white blood cells but no red blood cells or bacteria. Her urine pregnancy test is negative. Which disorder of the vulva is most likely in this case? · Genital herpes · Condylomata acuminata
  • 16. · Syphilitic chancre · Epidermoid cyst · Question Points: 0.0 / 1.0 · A 36-year-old married bank teller presents to your office, complaining of pain with defecation and occasional blood on the toilet paper. She states that last week she had food poisoning with nausea, vomiting, and diarrhea. She had runny stools but no black or bloody stools. Ever since her illness, she has continued to have severe pain with bowel movements. She now tries to put off defecation as long as possible. Although she is having constipation she denies any further diarrhea or leakage of stool. She has a past medical history of hypothyroidism and two spontaneous vaginal deliveries. She has had no other chronic illnesses or surgeries. She does not smoke and rarely drinks. She has two children. There is no family history of breast or colon cancer. She has had no weight gain, weight loss, fever, or night sweats. On examination she is afebrile, with a blood pressure of 115/70 and a pulse of 80. On abdominal examination, she has active bowel sounds, is nontender in all quadrants, and has no hepatosplenomegaly. Inspection of the anus reveals inflammation on the posterior side with erythema. Digital rectal examination is painful for the patient but no abnormalities are palpated. Anoscopic examination reveals no inflammation or bleeding. What is the anal disorder that best describes her symptoms? · Anorectal fistula · External hemorrhoids · Anal fissure
  • 17. · Anorectal cancer · Question Points: 0.0 / 1.0 · Adam is a very successful 15-year-old student and athlete. His mother brings him in today because he no longer studies, works out, or sees his friends. This has gone on for a month and a half. When you speak with him alone in the room, he states it “would be better if I was not here.” What would you do next? · Tell him that he has a very promising career in anything he chooses and soon he will feel better. · Tell him that he needs an antidepressant and it will take about 4 weeks to work. · Speak with his mother about getting him together more with his friends. · Assess his suicide risk. · Question Points: 1.0 / 1.0 · Which of the following is a “red flag” regarding patients presenting with headache? · Unilateral headache · Pain over the sinuses · New onset over age 40 · Aggravated or relieved by change in position
  • 18. · Question Points: 1.0 / 1.0 · A 73-year-old nurse presents to your office for evaluation of new onset of pill rolling tremors. She is not on any medications and does not take herbs or supplements. She has no chronic medical conditions. She does not smoke or drink alcohol. She walks into the examination room with slow movements and shuffling steps. She has decreased facial mobility and a blunt expression, without any changes in hair distribution on her face. Based on this description, what is the most likely reason for the patient's symptoms? · Cushing's syndrome · Nephrotic syndrome · Myxedema · Parkinson's disease · Question Points: 1.0 / 1.0 · Two weeks ago, Mary started a job which requires carrying 40-pound buckets. She presents with elbow pain worse on the right. On examination, it hurts her elbows to dorsiflex her hands against resistance when her palms face the floor. What condition does she have? · Medial epicondylitis (golfer's elbow) · Olecranon bursitis · Lateral epicondylitis (tennis elbow) · Supracondylar fracture
  • 19. · Question Points: 0.0 / 1.0 · A 26-year-old violinist presents to your clinic complaining of anxiety. He is a first-chair violinist in the local symphony orchestra and has started having symptoms during performances, such as sweating, shaking, and hyperventilating. It has gotten so bad that he has thought about giving up his first-chair status so he does not have to play the solo during one of the movements. He says that he never has these symptoms during rehearsals or when he is practicing. He denies having any of these symptoms at any other time. His past medical history is unremarkable. He denies any tobacco use, drug use, or alcohol abuse. His parents are both healthy. On examination you see a young man who appears worried. His vital signs and physical examination are unremarkable. What type of anxiety disorder best describes this situation? · Panic disorder · Specific phobia · Social phobia · Generalized anxiety disorder · Question Points: 0.0 / 1.0 · A 52-year-old secretary presents to your office, complaining about accidentally leaking urine when she coughs or sneezes. She says this has been going on for about a year now. She relates that she has not had a period for 2 years. She denies any recent illness or injuries. Her past medical history is significant for four spontaneous vaginal deliveries. She is married and has four children. She denies alcohol, tobacco, or drug use. During her pelvic examination, you note some atrophic vaginal tissue, but the remainder of her pelvic, abdominal, and rectal
  • 20. examinations are unremarkable. Which type of urinary incontinence does she have? · Stress incontinence · Urge incontinence · Overflow incontinence · Mixed Incontinence · Question Points: 0.0 / 1.0 · A 37-year-old nurse presents for evaluation of colicky right upper quadrant abdominal pain. The pain is associated with nausea and vomiting and occurs 1 to 2 hours after eating greasy foods. Which one of the following physical examination descriptions would be most consistent with the diagnosis of cholecystitis? · Abdomen is soft, nontender, and nondistended, without hepatosplenomegaly or masses. · Abdomen is soft and tender to palpation in the right lower quadrant, without rebound or guarding. · Abdomen is soft and tender to palpation in the right upper quadrant with inspiration, to the point of stopping inspiration, and there is no rebound or guarding. · Abdomen is soft and tender to palpation in the mid-epigastric area, without rebound or guarding. · Question Points: 0.0 / 1.0 · A 29-year-old computer programmer presents to your office
  • 21. for evaluation of a headache. The tightening sensation is located all over the head and is of moderate intensity. It used to last minutes, but this time it has lasted for 5 days. He denies photophobia and nausea. He spends several hours each day at a computer monitor/keyboard. He has tried over-the-counter medication; it has dulled the pain but not taken it away. Based on this description, what is your most likely diagnosis? · Tension · Migraine · Cluster · Analgesic rebound · Question Points: 1.0 / 1.0 · An 81 year old patient complains of shortness of breath for the past few days. On examination, you note late inspiratory crackles in the lower third of the chest and bilateral pedal edema that were not present a week ago. What is the most likely explanation for these? · Asthma · COPD · Bronchiectasis · Heart failure · Question Points: 1.0 / 1.0 · A high school soccer player “blew out his knee” when the opposing goalie's head and shoulder struck his flexed knee
  • 22. while the goalie was diving for the ball. All of the following structures were involved in some way in his injury, but which of the following is actually an extra-articular structure? · Synovium · Joint capsule · Juxta-articular bone · Tendons · Question Points: 1.0 / 1.0 · A woman in her 30th week has a cervical length estimated at 1 cm. Should you be concerned? · Yes; she may be at risk for preterm labor. · Yes; she most likely has a bicornuate uterus. · No; this is a normal measurement for this gestational age. · Yes; it likely indicates the fetus is in the breech position. · Question Points: 1.0 / 1.0 · A young woman presents in for a routine wellness examination. You notice that her vaginal walls have deep rugae and are slightly bluish in color. She also has a thicker white discharge. What should you suspect? · Hypoxia · Varicosities · Pregnancy
  • 23. · Sexually transmitted infection · Question Points: 1.0 / 1.0 · Ms. Whiting is a 68-year-old who presents in for her usual follow-up visit. You notice a few flat, red, and purple lesions, about 6 centimeters in diameter, on the ulnar aspect of her forearms but nowhere else. She doesn't mention them. They are tender when you examine them. What should you do? · Conclude that these are lesions she has had for a long time. · Wait for her to mention them before asking further questions. · Ask how she acquired them. · Conduct the visit as usual for the patient. · Question Points: 1.0 / 1.0 · A 58-year-old man presents to your office complaining of bilateral back pain that now awakens him at night. This has been steadily increasing for the past 2 months. Which one of the following is the most reassuring to you as a clinician in this patient with back pain? · Age over 50 · Pain at night · Pain lasting more than 1 month or not responding to therapy · Pain that is bilateral
  • 24. · Question Points: 1.0 / 1.0 · A 19-year-old childcare worker presents to you for her first prenatal visit. She cannot remember when her last period was but thinks it was between 2 and 5 months ago. When she began gaining weight and feeling “something” moving down there, she did a home pregnancy test and it was positive. She states she felt the movement about a week ago. She has had no nausea, vomiting, fatigue, or fevers. Her past medical history is remarkable only for irregular periods. She has been dating the same young man for a year. She says they were not using condoms. On examination, you see an overweight young lady appearing her stated age. Her head, eyes, ears, nose, throat, neck, thyroid, cardiac, … Test 40 QS · Question Points: 0.0 / 1.0 · Which of the following represents age-related changes in the lungs? · Decrease in chest wall compliance · Speed of expiration increases · Increase in respiratory muscle strength · Increased elastic recoil of lung tissue · Question Points: 0.0 / 1.0 · A 32-year-old white female presents to your clinic complaining of overwhelming sadness. She says for the past 2 months she has had crying episodes, difficulty sleeping, and problems with overeating. She says she used to go out with her friends from work, but now she just wants to go home and be by
  • 25. herself. She also thinks that her work productivity has been dropping because she just is too tired to care or concentrate. She denies any feelings of guilt or any suicidal ideation. She states that she has never felt this way in the past. She denies any recent illness or injuries. Her past medical history consists of an appendectomy when she was a teenager; otherwise, she has been healthy. She is single and works as a clerk in a medical office. She denies tobacco, alcohol, or illegal drug use. Her mother has high blood pressure and her father has had a history of mental illness. On examination, you see a woman appearing her stated age who seems quite sad. Her facial expression does not change while you talk to her and she makes little eye contact. She speaks so softly you cannot always understand her. Her thought processes and content seem unremarkable. What type of mood disorder do you think is most likely? · Dysthymic disorder · Manic (bipolar) disorder · Major depressive episode · Anxiety disorder · Question Points: 0.0 / 1.0 · A 40-year-old mother of two presents to your office for consultation. She is interested in knowing what her relative risks are for developing breast cancer. She is concerned because her sister had unilateral breast cancer 6 years ago at age 38. The patient reports on her history that she began having periods at age 11 and has been fairly regular ever since, except during her two pregnancies. Her first child arrived when she was 26 and her second at age 28. Otherwise, she has had no health problems. Her father has high blood pressure. Her mother had unilateral breast cancer in her 70s. The patient denies tobacco,
  • 26. alcohol, or drug use. She is a family law attorney and is married. Her examination is essentially unremarkable. Which risk factor of her personal and family history most puts her in danger of getting breast cancer? . · Age at menarche of less than 12 · First live birth between the ages of 25 and 29 · · First-degree relative with postmenopausal breast cancer · First-degree relative with premenopausal breast cancer · Question Points: 0.0 / 1.0 · Ray works a physical job and notes pain when he attempts to lift his arm over his head. When you move the shoulder passively, he has full range of motion without pain and there is no gross swelling or tenderness. What type of joint disease does this most likely represent? · Articular · Extra-articular · Neither · Both · Question Points: 0.0 / 1.0 · A 36-year-old married bank teller presents to your office, complaining of pain with defecation and occasional blood on the toilet paper. She states that last week she had food poisoning with nausea, vomiting, and diarrhea. She had runny stools but no black or bloody stools. Ever since her illness, she has continued to have severe pain with bowel movements. She
  • 27. now tries to put off defecation as long as possible. Although she is having constipation she denies any further diarrhea or leakage of stool. She has a past medical history of hypothyroidism and two spontaneous vaginal deliveries. She has had no other chronic illnesses or surgeries. She does not smoke and rarely drinks. She has two children. There is no family history of breast or colon cancer. She has had no weight gain, weight loss, fever, or night sweats. On examination she is afebrile, with a blood pressure of 115/70 and a pulse of 80. On abdominal examination, she has active bowel sounds, is nontender in all quadrants, and has no hepatosplenomegaly. Inspection of the anus reveals inflammation on the posterior side with erythema. Digital rectal examination is painful for the patient but no abnormalities are palpated. Anoscopic examination reveals no inflammation or bleeding. What is the anal disorder that best describes her symptoms? · Anorectal fistula · External hemorrhoids · Anal fissure · Anorectal cancer · Question Points: 0.0 / 1.0 · Adam is a very successful 15-year-old student and athlete. His mother brings him in today because he no longer studies, works out, or sees his friends. This has gone on for a month and a half. When you speak with him alone in the room, he states it “would be better if I was not here.” What would you do next? · Tell him that he has a very promising career in anything he chooses and soon he will feel better. · Tell him that he needs an antidepressant and it will take about 4 weeks to work.
  • 28. · Speak with his mother about getting him together more with his friends. · Assess his suicide risk. · Question Points: 1.0 / 1.0 · Two weeks ago, Mary started a job which requires carrying 40-pound buckets. She presents with elbow pain worse on the right. On examination, it hurts her elbows to dorsiflex her hands against resistance when her palms face the floor. What condition does she have? · Medial epicondylitis (golfer's elbow) · Olecranon bursitis · Lateral epicondylitis (tennis elbow) · Supracondylar fracture · Question Points: 0.0 / 1.0 · A 26-year-old violinist presents to your clinic complaining of anxiety. He is a first-chair violinist in the local symphony orchestra and has started having symptoms during performances, such as sweating, shaking, and hyperventilating. It has gotten so bad that he has thought about giving up his first-chair status so he does not have to play the solo during one of the movements. He says that he never has these symptoms during rehearsals or when he is practicing. He denies having any of these symptoms at any other time. His past medical history is unremarkable. He denies any tobacco use, drug use, or alcohol abuse. His parents are both healthy. On examination you
  • 29. see a young man who appears worried. His vital signs and physical examination are unremarkable. What type of anxiety disorder best describes this situation? · Panic disorder · Specific phobia · Social phobia · Generalized anxiety disorder · Question Points: 0.0 / 1.0 · A 52-year-old secretary presents to your office, complaining about accidentally leaking urine when she coughs or sneezes. She says this has been going on for about a year now. She relates that she has not had a period for 2 years. She denies any recent illness or injuries. Her past medical history is significant for four spontaneous vaginal deliveries. She is married and has four children. She denies alcohol, tobacco, or drug use. During her pelvic examination, you note some atrophic vaginal tissue, but the remainder of her pelvic, abdominal, and rectal examinations are unremarkable. Which type of urinary incontinence does she have? · Stress incontinence · Urge incontinence · Overflow incontinence · Mixed Incontinence · Question Points: 0.0 / 1.0
  • 30. · A 37-year-old nurse presents for evaluation of colicky right upper quadrant abdominal pain. The pain is associated with nausea and vomiting and occurs 1 to 2 hours after eating greasy foods. Which one of the following physical examination descriptions would be most consistent with the diagnosis of cholecystitis? · Abdomen is soft, nontender, and nondistended, without hepatosplenomegaly or masses. · Abdomen is soft and tender to palpation in the right lower quadrant, without rebound or guarding. · Abdomen is soft and tender to palpation in the right upper quadrant with inspiration, to the point of stopping inspiration, and there is no rebound or guarding. · Abdomen is soft and tender to palpation in the mid-epigastric area, without rebound or guarding. · Question Points: 0.0 / 1.0 · A 29-year-old computer programmer presents to your office for evaluation of a headache. The tightening sensation is located all over the head and is of moderate intensity. It used to last minutes, but this time it has lasted for 5 days. He denies photophobia and nausea. He spends several hours each day at a computer monitor/keyboard. He has tried over-the-counter medication; it has dulled the pain but not taken it away. Based on this description, what is your most likely diagnosis? · Tension · Migraine · Cluster
  • 31. · Analgesic rebound · Question Points: 0.0 / 1.0 · You are having trouble examining the abdomen of a school- aged child due to ticklishness. What should you do? · Have the child press on your hand. · Have the parent insist that the child allow you to examine her. · Ask the parent to leave the room. · Make the child realize that this is part of the examination and must be done. · Question Points: 1.0 / 1.0 · A 27-year-old woman is brought to your office by her mother. The mother tells you that her daughter has been schizophrenic for the last 8 years and is starting to decompensate despite medication. The patient states that she has been taking her antipsychotic and she is doing just fine. Her mother retorts that her daughter has become quite paranoid. When asked why, the mother gives an example about the mailman. She says that her daughter goes and gets the mail every day and then microwaves the letters. The patient agrees that she does this but only because she sees the mailman flipping through the envelopes and she knows he's putting anthrax on the letters. Her mother turns to her and says, “He's only sorting the mail!” Which best describes the patient's abnormality of perception? · Illusion · Hallucination · Fugue state
  • 32. · Facticious · Question Points: 0.0 / 1.0 Top of Form · A 29-year-old married computer programmer presents to your clinic, complaining of “something strange” going on in his scrotum. Last month while he was doing his testicular self- examination, he felt a lump in his left testis. He waited a month and felt the area again, but the lump was still there. He has had some aching in his left testis but denies any pain with urination or sexual intercourse. He denies any fever, malaise, or night sweats. His past medical history consists of groin surgery when he was a baby and a tonsillectomy as a teenager. He eats a healthy diet and works out at the gym five times a week. He denies any tobacco or illegal drugs and drinks alcohol occasionally. His parents are both healthy. On examination, you see a muscular, healthy, young-appearing man with unremarkable vital signs. On visualization, the penis is circumcised with no lesions; there is a scar in his right inguinal region. There is no lymphadenopathy. Palpation of his scrotum is unremarkable on the right but indicates a large mass on the left. Placing a finger through the inguinal ring on the right, you have the patient bear down. Nothing is felt. You attempt to place your finger through the left inguinal ring but cannot get above the mass. On rectal examination, his prostate is unremarkable. What disorder of the testes is most likely the diagnosis? · Hydrocele · Inguinal hernia · Scrotal edema
  • 33. · Varicocele · Question Points: 1.0 / 1.0 Bottom of Form · A high school football player injured his wrist in a game. He is tender between the two tendons at the base of the thumb. Which of the following should be considered? · DeQuervain's tenosynovitis · Scaphoid fracture · Wrist sprain · Rheumatoid arthritis · Question Points: 0.0 / 1.0 · An adolescent male presents to your clinic with a note from his mother stating it is okay for him to be seen today without her presence. He has come in for his annual sports physical required to play football. For his age his physical examination is unremarkable, and you sign his school's physical examination form. You decide to take this opportunity to do some health education with him. He admits to wondering a lot lately if he is normal. Although he is in football he really enjoys science and computers more. He is worried that all his buddies will think he is a geek. He is convinced he also won't get a date for the Sadie Hawkins dance next week because the girls all think he is boring, too. He denies any experimentation with tobacco or alcohol, and he blushes when you mention sex. After hitting all the pertinent age-appropriate education points, you give him his sports physical form and he leaves. The patient's concerns during the visit most resemble what developmental stage of adolescence?
  • 34. · Early adolescence (10 to 14 years old) · Middle adolescence (15 to 16 years old) · Late adolescence (17 to 20 years old) · Adulthood · Question Points: 0.0 / 1.0 · You are observing an infant who is able to pull to a stand, uses “mama” and “dada” specifically, and indicates his wants by vocalization and pointing. Where would you place this child's developmental age? · 12 months · 10 months · 8 months · 6 months · Question Points: 0.0 / 1.0 · A 77-year-old retired school superintendent presents to your office, complaining of unsteady hands. He says that, for the past 6 months, when his hands are resting in his lap they shake uncontrollably. He says when he holds them out in front of his body the shaking diminishes, and when he uses his hands the shaking is also better. He also complains of some difficulty getting up out of his chair and walking around. He denies any recent illnesses or injuries. His past medical history is significant for high blood pressure and coronary artery disease, requiring a stent in the past. He has been married for more than
  • 35. 50 years and has five children and 12 grandchildren. He denies any tobacco, alcohol, or drug use. His mother died of a stroke in her 70s, and his father died of a heart attack in his 60s. He has a younger sister who has arthritis problems. His children are all essentially healthy. On examination, you see a fine, pill-rolling tremor of his left hand. His right shows less movement. His cranial nerve examination is normal. He has some difficulty rising from his chair, his gait is slow, and it takes him time to turn around to walk back toward you. He has almost no “arm swing” with his gait. What type of tremor is he most likely to have? · Resting tremor · Postural tremor · Intention tremor · Drug Induced Tremor · Question Points: 1.0 / 1.0 · A 35-year-old stockbroker presents to your office, complaining of feeling tired and irritable. She also says she feels like nothing ever goes her way and that nothing good ever happens. When you ask her how long she has felt this way she laughs and says, “Since when have I not?” She relates that she has felt pessimistic about life in general since she was in high school. She denies any problems with sleep, appetite, or concentration, and states she hasn't thought about killing herself. She reports no recent illnesses or injuries. She is single. She smokes one pack of cigarettes a day, drinks occasionally, and hasn't taken any illegal drugs since college. Her mother suffers from depression and her father has high blood pressure. On examination her vital signs and physical examination are unremarkable.
  • 36. What mental health disorder best describes her symptoms? · Major depressive episode · Dysthymic disorder · Cyclothymic disorder · Hypothryoidism · Question Points: 0.0 / 1.0 · Alexandra is a 28-year-old editor who presents to the clinic with abdominal pain. The pain is a dull ache, located in the right upper quadrant, that she rates as a 3 at the least and an 8 at the worst. The pain, which started a few weeks ago, comes and goes, lasts for 2 to 3 hours at a time, and seems to be worse a couple of hours after eating. She has noticed that it starts after eating greasy foods, so she has cut down on these as much as she can. Initially, it occurred once a week, but now it is occurring every other day. Nothing makes it better. From this description, which of the seven attributes of a symptom has been omitted? · Setting in which the symptom occurs · Associated manifestations · Quality · Timing · Question Points: 1.0 / 1.0 · Susane is a 27-year-old who has had headaches, muscle aches, and fatigue for the last 2 months. You have completed a
  • 37. thorough history, examination, and laboratory workup but have not found a cause. What would be an appropriate next course of action? · A referral to a neurologist · A referral to a rheumatologist · To tell the patient you can't find anything · To screen for depression · Question Points: 0.0 / 1.0 · A 73-year-old retired accountant presents to your office for her annual examination. She has incontinence of urine when she coughs or sneezes. She takes several medications for control of hypertension and diabetes. You use the DIAPPERS mnemonic to assess the cause of her incontinence. All of the following are items represented by the mnemonic except for: · Atrophic vaginitis · Depression · Pharmaceuticals · Restricted mobility · Question Points: 1.0 / 1.0 · A 8-year-old patient presents to the office for evaluation of a rash. At first, there was only one large patch, but then more lesions erupted suddenly on the back and torso; the lesions itch. On physical examination, you note that the pattern of eruption is like a Christmas tree and that there are a variety of
  • 38. erythematous papules and macules on the cleavage lines of the back. Based on this description, what is the most likely diagnosis? · Pityriasis rosea · Tinea versicolor · Psoriasis · Atopic eczema · Question Points: 0.0 / 1.0 · A 79-year-old retired banker presents to your office for evaluation of difficulty with urination; he gets up five to six times per night to urinate and has to go at least that often in the daytime. He does not feel as if his bladder empties completely; the strength of the urinary stream is diminished. He denies dysuria or hematuria. This problem has been present for several years but has worsened over the last 8 months. You palpate his prostate. What is your expected physical examination finding, based on this description? · Normal size, smooth · Normal size, boggy · Enlarged size, smooth · Enlarged size, boggy · Question Points: 1.0 / 1.0 · A 49-year-old administrative assistant presents to your office for evaluation of dizziness. You elicit the information that the
  • 39. dizziness is a spinning sensation of sudden onset, worse with head position changes. The episodes last a few seconds and then go away, and they are accompanied by intense nausea. She has vomited one time. She denies tinnitus. You perform a physical examination of the head and neck and note that the patient's hearing is intact. Pt has normal results of Weber and Rinne and that there is nystagmus. Her gait is normal. Based on this description, what is the most likely diagnosis? · Benign paroxysmal positional vertigo · Vestibular neuronitis · Ménière's disease · Acoustic neuroma · Question Points: 0.0 / 1.0 · An 88-year-old retired piano teacher presents for evaluation of fatigue. You notice that her clothes are hanging loosely off her frame and that she has lost 15 pounds. She is unaware of this. Her husband of 63 years died a few months ago. You ask the patient to complete a Rapid Screen for Dietary Intake. Which of the following statements is considered to be part of this rapid screen? · I eat more than two meals per day. · I drink one glass of alcohol every day. · Without wanting to, I have lost or gained 10 pounds in the last 6 months. · I eat with at least one other person most of the time. ·
  • 40. Question Points: 1.0 / 1.0 · A 75-year-old homemaker brings her 76-year-old husband to your clinic. She states that 4 months ago he had a stroke and ever since she has been frustrated with his problems with communication. They were at a restaurant after church one Sunday when he suddenly became quiet. When she realized something was wrong he was taken to the hospital by EMS. He spent 2 weeks in the hospital with right-sided weakness and difficulty speaking. After hospitalization he was in a rehab center, where he regained the ability to walk and most of the use of his right hand. He also began to speak more, but she says that much of the time “he doesn't make any sense.” She gives an example that when she reminded him the car needed to be serviced he told her “I will change the Kool-Aid out of the sink myself with the ludrip.” She says that these sayings are becoming frustrating. She wants you to tell her what is wrong and what you can do about it. While you write up a consult to neurology, you describe the syndrome to her. · Wernicke's aphasia · Broca's aphasia · Dysarthria · Dementia · Question Points: 0.0 / 1.0 · A 12-year-old presents to the clinic with his father for evaluation of a painful lump at the left eyelid. It started this morning. He denies any trauma or injury. There is no visual disturbance. Upon physical examination, there is a red raised area at the margin of the eyelid that is tender to palpation; no tearing occurs with palpation of the lesion. Based on this
  • 41. description, what is the most likely diagnosis? · Dacryocystitis · Chalazion · Hordeolum · Xanthelasma · Question Points: 0.0 / 1.0 · A young man feels something in his scrotum and presents to you for clarification. On your examination, you note what feels like a “bag of worms” in the left scrotum, superior to the testicles. Which of the following is most likely? · Hydrocele of the spermatic cord · Varicocele · Testicular carcinoma · A normal vas deferens · Question Points: 0.0 / 1.0 · A 28-year-old graduate student presents to your clinic for evaluation of pain “all over.” With further questioning, she is able to relate that the pain is worse in the neck, shoulders, hands, lower back, and knees. She denies swelling in her joints; she states that the pain is worse in the morning; there is no limitation in her range of motion. On physical examination, she has several points on the muscles of the neck, shoulders, and back that are tender to palpation; muscle strength and range of motion are normal. Which of the following is likely the cause of
  • 42. her pain? · Rheumatoid arthritis · Osteoarthritis · Fibromyalgia · Polymyalgia rheumatic · Question Points: 0.0 / 1.0 · Jim is a 60-year-old man who presents with vomiting. He denies seeing any blood with emesis, which has been occurring for 2 days. He does note a dark, granular substance resembling coffee grounds. What do you suspect? · Bleeding from a diverticulum · Bleeding from a peptic ulcer · Bleeding from a colon cancer · Bleeding from cholecystitis · Question Points: 1.0 / 1.0 · The following information is recorded in the health history: “I feel really tired.” To which category does this information belong? · Chief complaint · Present illness · Personal and social history
  • 43. · Review of systems · Question Points: 0.0 / 1.0 · A 24-year-old secretary presents to your clinic complaining of difficulty sleeping, severe nightmares, and irritability. She states it all began 6 months ago when she went to a fast-food restaurant at midnight. While she was waiting in her car, a man entered through the passenger door and put a gun to her head. He had her drive to a remote area where he took her money and threatened to kill her. When the gun jammed, he panicked and ran off. Ever since this occurred, the patient has been having these symptoms. She states she jumps at every noise and refuses to drive at night. She states her anxiety has had such a marked influence on her job performance that she is afraid she will be fired. She denies any recent illnesses or injuries. Her past medical history is unremarkable. On examination you find a nervous woman appearing her stated age. Her physical examination is unremarkable. You recommend medication and counseling. What anxiety disorder do you think this young women most likely has? · Specific phobia · Acute stress disorder · Post-traumatic stress disorder · Generalized anxiety disorder · Question Points: 1.0 / 1.0 · A 41-year-old real estate agent presents to your office, complaining that he feels like his face is paralyzed on the left.
  • 44. He states that last week he felt his left eyelid was drowsy and as the day progressed he was unable to close his eyelid all the way. Later he felt like his smile became affected also. He denies any recent injuries but had an upper respiratory viral infection last month. His past medical history is unremarkable. He is divorced and has one child. He smokes one pack of cigarettes a day, occasionally drinks alcohol, and denies any illegal drug use. His mother has high blood pressure, and his father has sarcoidosis. On examination, you ask him to close his eyes. He is unable to close his left eye. You ask him to open his eyes and raise his eyebrows. His right forehead furrows but his left remains flat. You then ask him to give you a big smile. The right corner of his mouth raises but the left side of his mouth remains the same. What type of facial paralysis does he have? · Peripheral CN VII paralysis · Central CN VII paralysis · CN V Neuralgia · CN V paralysis · Question Points: 0.0 / 1.0 · Which of the following booster immunizations is recommended in the older adult population? · Tetanus · Diphtheria · Measles · Mumps
  • 45. · Question Points: 0.0 / 1.0 · A 26-year-old white female presents to your clinic at 38 weeks, complaining of intermittent contractions. They last for 30 seconds and are coming every 10 minutes. Her prenatal course has so far been uneventful. You send her to labor and delivery for a labor assessment. On vaginal examination, she has effaced 4 cm, but you cannot feel a presenting part. You admit her for active labor; however, you wish to assess if she is vertex (baby's head is down), so you do the Leopold's maneuver. Palpating the upper pole with your hands, you feel a firm round mass. Placing your hand along the right side of her abdomen, you feel a smooth firmness. Palpating your other hand along the left side of her abdomen, you feel irregular bumps. Above the pelvic brim you feel a firm irregular mass. While awaiting ultrasound to confirm your diagnosis, you write the pertinent orders. How is this fetus presenting? · Vertex · Breech · Transverse · Anteverted · Question Points: 1.0 / 1.0 · A 29-year-old homemaker who is G4P3 presents to your clinic for her first prenatal check. Her last period was 2 months ago. She has had three previous pregnancies and deliveries with no complications. She has no medical problems and has had no surgeries. Her only current complaint is of severe reflux that occurs in the mornings and evenings. On examination she is in no acute distress. Her vitals are 110/70 with a pulse of 88. Her respirations are 16. Her head, eyes, ears, nose, throat, thyroid,
  • 46. cardiac, pulmonary, and abdominal examinations are unremarkable. On bimanual examination her cervix is soft and her uterus is 10 weeks in size. Pap smear, cultures, and blood work are pending. What is the most likely cause of her first-trimester reflux? · Increasing prolactin levels · Increasing ADH (antidiuretic hormone) levels · Increasing progesterone · Enlarged gravid uterus · Question Points: 0.0 / 1.0 · A patient is assigned a visual acuity of 20/100 in her left eye. Which of the following is true? · She obtains a 20% correct score at 100 feet. · She can accurately name 20% of the letters at 20 feet. · She can see at 20 feet what a normal person could see at 100 feet. · She can see at 100 feet what a normal person could see at 20 feet. · Question Points: 1.0 / 1.0 · A 72-year-old African-American male is brought to your clinic by his daughter for a follow-up visit after his recent hospitalization. He had been admitted to the local hospital for speech …